SlideShare una empresa de Scribd logo
1 de 53
SOLUCIONES Y PROPIEDADES COLIGATIVAS


EXIJA SU LIBRO




3.1 DEFINICIÓN DE SOLUCIÓN


M      uy pocos materiales que encontramos en la vida diaria son sustancias puras; la mayor
       parte son mezclas y muchas de ellas son homogéneas, Recordemos que las mezclas
homogéneas se denominan soluciones o disoluciones, por ejemplo: el aire que respiramos es
una mezcla homogénea de varias sustancias gaseosas. El latón es una solución de cinc en
cobre. Los océanos son una solución de muchas sustancias que se han disuelto en agua.

               Una solución es una mezcla homogénea de dos o mas sustancias puras,
               denominadas componentes de la solución, las cuales pueden ser gaseosas,
líquidas o sólidas; debido a que las soluciones líquidas son las mas comunes, en esta sección
enfocaremos nuestro estudio en dicho estado. Sin embargo, el estado físico de una solución lo
determina a menudo el estado de su componente principal, denominado disolvente. El otro u
otros componentes se denominan soluto.

                                                            Soluto es la sustancia que se
                   Disolvente o solvente, es la sustancia
                                                            disuelve a través del disolvente en
                   donde se disuelve el llamado soluto.
                                                            forma de moléculas o iones para
                   Generalmente es aquella que se
                                                            formar una sola fase. En una
                   encuentra en mayor cantidad. En toda
                                                            solución puede existir mas de un
                   solución existe un solo solvente.
                                                            soluto.




                   AGUA
           YUPI




  COMPENDIOS DE QUÍMICA GENERAL PARA ESTUDIANTES DE CIENCIAS E INGENIERÍA
58                      SOLUCIONES Y PROPIEDADES COLIGATIVAS
En la figura 3.1 es un resumen de la relación entre los materiales (materia), recuerde que las
sustancias pueden ser mezcladas por procesos físicos y se pueden usar otros procesos físicos
para separar la mezcla en 1sustancias.

3.2 PSEUDO - SOLUCIÓN, SUSPENSIÓN Y                                                     materia
COLOIDE


C    uando una sustancia se disuelve o dispersa a
     través de otra, formaremos una mezcla donde
encontramos tres posibilidades diferentes de
                                                                      sustancias



                                                                                          Mezclas
                                                                                                       mezclas




tamaños de partículas. Cada uno de estos casos                 elementos                homogéneas
                                                                                        (soluciones)
dará lugar a mezclas denominadas: solución,
coloide y suspensión, con propiedades diferentes.                                                          Mezclas
                                                                           compuestos                    heterogéneas
La tabla 4.1 muestra una clasificación según el
tamaño partículas.
                                                                              Figura 3.1
                                                                              Relación entre los materiales

     1. Cuando la sustancia se disuelve en forma de moléculas o iones entonces se denomina
        solución y los componentes son denominados soluto y disolvente.
     2. Cuando la sustancia se dispersa (no es soluble) y permanece firmemente dividida se
        denomina coloide y sus componentes son llamados fase dispersa y fase dispersante.
     3. Cuando el tamaño de la sustancia a quién se le denomina realmente micela, es mas
        grande en comparación a los dos casos anteriores se denominará suspensión.
                       TABLA 3.1
                       Clasificación de la mezclas según el tamaño de miscela


                                      SOLUCION       COLOIDE        SUSPENSIÓN

                    TAMAÑO DE
                     MISCELAS            1-10Å       10-10000Å         >10000Å



3.2.1 Coloides hidrofílicos e hidrofóbicos


C    oloide es una pseudo – solución, es una mezcla heterogénea en la cual las partículas del
     soluto no precipitan, se dispersan a través de una fase dispersante, se clasifican en dos
clases principales: coloides hidrofílicos y coloides hidrofóbicos. Un coloide hidrofílico (que
aman el agua) es un coloide en el cual hay una atracción fuerte entre la fase dispersa y la fase
continua (agua). Muchos de estos coloides consisten en macromoléculas (moléculas muy
grandes) dispersas en agua. Excepto por el gran tamaño de las moléculas dispersas, estos
coloides parecen soluciones normales. Un coloide hidrofóbico (que rechazan el agua), es un
coloide en el cual hay una falta de atracción entre la fase dispersa y la fase continua (agua)
Los coloides hidrofóbicos son básicamente inestables.


1
 1 Ebbing Darrell D. Según este autor una sustancia (o sustancia pura) es una clase de materia que no puede ser
separada en otras clases de materia por ningún proceso físico.

    COMPENDIOS DE QUÍMICA GENERAL PARA ESTUDIANTES DE CIENCIAS E INGENIERÍA
SOLUCIONES Y PROPIEDADES COLIGATIVAS                                  59
La tabla 3.2 muestra una clasificación de 8 tipos de coloides que son producto de la mezcla
entre los estados sólido, líquido y gaseoso, pero respetando el orden para identificar a la fase
dispersa y fase dispersante. No se incluye la mezcla gas-gas porque estos se componen de
moléculas individuales.



                     TABLA 3.2
                     Clasificación de los coloides según el estado de agregación

                  FASE            FASE                  TIPO DE            EJEMPLO
                DISPERSA     DISPERSANTE              COLOIDE
               Gas           Gas                 .................   ..................
               Líquido       Gas                 Aerosol líquido     Niebla
               Sólido        Gas                 Aerosol sólido      Humo
               Gas           Líquido             Espuma              Espuma de cerveza
               Líquido       Líquido             Emulsión            Leche
               Sólido        Líquido             Sol y gel           Pintura, jabón en agua
               Gas           Sólido              Espuma sólida       Esponja, piedra pómez
               Líquido       Sólido              Emulsión sólida     Mantequilla
               Sólido        Sólido              Sol sólida          Porcelana

Una de las formas prácticas que permite identificar a los coloides, es el hecho de aprovecha el
efecto Tyndall, donde se refleja la luz mucho mas intensa en una suspensión que en un
coloide, en una solución no hay dicho reflejo.

3.3 CLASIFICACIÓN                DE      LAS                                      Haz de luz no visible
SOLUCIONES


L   as soluciones se pueden clasificar,
    atendiendo a 5 aspectos importantes:
                                                   Fuente luminosa

                                                           Haz de luz no
                                                           visible
                                                                              A           B           C


3.3.1 Según el número de componentes

    Soluciones binarias: de dos
                                                                     Haz de luz visible       Haz de luz visible
     componentes
                                                     Figura 3.2
    Soluciones ternarias: de tres                   Dispersión de un haz de luz por partículas
     componentes                                     coloidales conocido como efecto Tyndall. La
    Soluciones cuaternarias: de cuatro              presencia de partículas coloidales pueden
     componentes                                     detectarse con facilidad con ayuda de un haz de
                                                     luz.
                                                     A: Oro coloidal, una solución dorada
3.3.2 Según la naturaleza del disolvente             B: Solución de NaCl
                                                     C: Humo de tabaco, Un aerosol
      Soluciones acuosas: el disolvente
       es el agua
      Soluciones orgánicas: El disolvente puede ser: benceno, acetona, etc

3.3.3 Según la naturaleza del soluto

    Soluciones ácidas: presencia de sustancias ácidas: HCl, HNO3, etc.
    Soluciones básicas: presencia de sustancias básicas: NaOH, Ca(OH)2
  COMPENDIOS DE QUÍMICA GENERAL PARA ESTUDIANTES DE CIENCIAS E INGENIERÍA
60                      SOLUCIONES Y PROPIEDADES COLIGATIVAS
      Soluciones neutras: presencia de sales neutras: NaCl, KNO3, etc.

3.3.4 De acuerdo a la cantidad de sus componentes

         Soluciones diluidas: Cuando la masa del soluto es pequeña con respecto al
          volumen total de la solución.
         Soluciones concentradas: Es aquella donde la cantidad del soluto es grande
          respecto al volumen total de la solución.
         Soluciones saturadas: Es aquella solución que ha alcanzado su máxima
          concentración a una temperatura determinada, es decir que no admite mas soluto
          porque este comienza a precipitar.
         Soluciones sobresaturadas: Es cuando se disuelve mas soluto que la solución
          saturada debido a ciertos factores mecánicos, por ejemplo la agitación donde se
          encuentra en equilibrio inestable.

3.3.5 Según los estados de agregación de la materia

      Soluciones sólidas: Donde sus componentes se hallan en el estado sólido.
      Soluciones liquidas: Donde sus componentes están estado líquido.
      Soluciones gaseosas: Donde sus componentes están en estado gaseoso.

Ejemplo 3.1.- Dar un ejemplo de una solución
constituida por las siguientes fases a) gas-gas, b)
líquido-gas, c) líquido-líquido, d) líquido-sólido, e)
sólido-sólido.
                                                         Solubilidad en g soluto/100 g agua




Solución:
            a) Gas- gas :             Aire (O2, CO2,
               etc en N2)
            b) Líquido – gas:         Soda ( CO2 (g)
               en H2O ())
            c) Líquido – líquido:     alcohol   para
               fricciones (H2O () en i-C3H7OH ())
            d) Líquido – sólido:      agua de mar
               (NaCl, LiCl, etc en H2O ())                                                                                       KI
                                                                                              200
            e) Sólido-sólido:         “oro” de 14
                                                                                              180
               quilates (Cu en Au)
                                                                                              160                               KNO3
3.4 SOLUBILIDAD                                                                               140


L
                                                                                              120
     a solubilidad se define como la cantidad de una                                                                              KBr
                                                                                              100
     sustancia que se disuelve en una cantidad dada
de disolvente (por ejemplo agua) a una temperatura                                             80
                                                                                                                                  KCl
dada para dar una solución saturada; es la                                                     60
concentración máxima posible. Por ejemplo, la sal                                              40                               NaCl
de cocina tiene una solubilidad de 36 g por 100 g de                                           20                               K2SO4
agua; por lo tanto, al añadir 40 gramos de NaCl,
quedaran 3 g de sal sin disolverse.                                                                 10 20 30 40 50 60 70 80 90 100
                                                                                                         Temperatura en grados Celcius

  COMPENDIOS DE QUÍMICA GENERAL PARA ESTUDIANTES 3.3 CIENCIAS E INGENIERÍA
                                           Figura DE
                                           Efecto de la temperatura en las solubilidades de algunas
                                           sales.
SOLUCIONES Y PROPIEDADES COLIGATIVAS                                          61
3.4.1 Factores que afectan la solubilidad


L   a solubilidad de un sólido es muy sensible a los cambios de temperatura; pero los
    cambios ordinarios de presión no producen ninguna variación significativa. La figura 3.3
muestra cómo la temperatura afecta la solubilidad de algunos sólidos corrientes en agua, por
ejemplo la sacarosa, el nitrato de amonio y el hidróxido de potasio ilustran muy bien este
fenómeno; por el contrario, la solubilidad del NaCl casi no se ve afectada por un cambio de
temperatura. En algunos casos es posible lograr concentraciones superiores a las
concentraciones de saturación, y entonces se tiene una solución sobresaturada. Una solución
sobresaturada se puede producir enfriando una solución concentrada caliente.

Los compuestos iónicos son solubles en agua, alcohol metílico y amoniaco líquido; e
insolubles en octano, benceno y tetracloruro de carbono. Las moléculas de agua, alcohol
                                          metílico y amoniaco son polares; cada molécula
       δ-                        δ-
                                          posee centros positivos y negativos de carga
       O              H          O
   δ+       δ +
                                    δ+    eléctrica como muestra la figura 3.4. Otros
   H        H                C      H     líquidos polares también actúan eficientemente
     AGUA                                 en la disolución de compuestos iónicos. Las
                       H
                                  H       moléculas de octano y benceno son no polares
           N         ALCOHOL METILICO     (ver     figura     3.5).    Puesto    que     las
                                          electronegatividades del carbono y del hidrógeno
     H             H                      es muy pequeña, cualquier enlace entre estos dos
           H      AMONIACO                átomos es no polar. El enlace entre el cloro y el
                                          carbono sí es polar, como se puede deducir al
       Figura 3.4                         estudiar las 2electronegatividades de la tabla 1.3
       Algunas moléculas polares          del capítulo 1 (página 20)

Por regla general los líquidos no polares son ineficientes en la disolución de compuestos
iónicos. También encontramos que un líquido polar disolverá otros compuestos polares. El
amoniaco y el alcohol metílico se disuelven en agua. La sacarosa, sólido polar (no iónico), es
soluble en agua y alcohol metílico, e insoluble en benceno, octano y tetracloruro de carbono.
Los disolventes no polares disuelven compuestos no polares. Las grasas y aceites,
hidrocarburos no polares, se disuelven en cualquiera de los tres solventes no polares que
hemos discutido previamente, pero son
virtualmente insolubles en los tres disolventes
polares. La regla que se sigue en la elección de          H                    Cl
disolventes dice:     “ sustancias similares              C
disuelven     sustancias     similares”.    Los    H C        C H               C
disolventes polares disuelven compuestos
iónicos y polares; y los no polares disuelven      H C         C H       Cl           Cl
compuestos no polares.                                    C                    Cl
                                                                    H
3.4.2 Solubilidad de los gases                                                      TETRACLORURO
                                                               BENCENO               DE CARBONO


L   os gases son muy poco solubles en agua y
    otros líquidos corrientes. La solubilidad de
un gas puede acrecentar aumentando la presión
                                                               Figura 3.5
                                                               Algunas moléculas no polares

2
  La electronegatividad de un elemento mide la tendencia relativa del átomo a atraer los electrones hacia sí
cuando se combina químicamente con otro átomo ( véase capítulo 1 ).
   COMPENDIOS DE QUÍMICA GENERAL PARA ESTUDIANTES DE CIENCIAS E INGENIERÍA
62                      SOLUCIONES Y PROPIEDADES COLIGATIVAS
sobre el gas que se halla por encima de la solución. La solubilidad del oxígeno a diferentes
presiones aparece en la tabla 4.3. Es obvio que al observar la última columna, que la
solubilidad es directamente proporcional a la presión del gas. Este comportamiento es normal
para soluciones de gases en líquidos.

        TABLA 3.3
        Solubilidad del oxígeno en agua a 25 ºC
  P [mmHg]       Solubilidad, [moles O2 m               La solubilidad de los gases en los
                 por litro de H2O]        ∗10 6 = constante
                                                        líquidos, al contrario de la de los
                                        P
    175                   0.000307             1.75     sólidos o líquidos, disminuye a medida
    300                   0.000500             1.67     que aumenta la temperatura. Un
    414                   0.000688             1.66     químico pondrá a hervir una muestra
    610                    0.00100             1.64     de agua para reducir la concentración
    760                    0.00128             1.68     de gases atmosféricos disueltos. Puesto
que la solubilidad del oxígeno en agua se reduce considerablemente con el calor, al vaciar
agua caliente en los ríos y lagos puede causar serio perjuicio a la vida acuática.

TABLA 3.4: Efecto de la temperatura sobre la
         solubilidad• de los gases en agua••
Gas    0 ºC        10 ºC      50 ºC      100 ºC      • La solubilidad se expresa en moles del gas disueltas por
H2     0.000960    0.000873   0.000717   0.000715    litro de agua
N2     0.00105     0.000830   0.000485   0.000423    •• La presión del gas encima de la solución es 1 atm.
O2     0.00212     0.00170    0.000933   0.000758
                                            El efecto de la temperatura sobre la solubilidad
CO2 0.0765         0.0533     0.0194
                                            de los gases se muestra en la información de la
tabla 3.4. La solubilidad del CO2 es mucho mayor que la de los otros tres gases, debido a que
el CO2 reacciona con el agua para forma ácido carbónico:

                                  CO2 + H2O ⇒ H2CO3

3.5 EL PROCESO DE DISOLUCIÓN


U    na sustancia puede disolverse con o sin reacción química en el disolvente. El sodio
     metálico se disuelve en agua desprendiendo burbujas de hidrógeno y cantidades
considerables de calor, se produce un cambio químico en el cual el H2 y el hidróxido de sodio
iónico soluble, NaOH, son los productos, la ecuación iónica total será:

                                       2Na(s) + 2H2O → 2[Na+(ac) + OH-(ac)] + H2(g)                         (1)

El cloruro de sodio sólido, por otra parte, se disuelve en agua sin evidencia de reacción
química:
                                               NaCl(s) → Na+(ac) + Cl-(ac)                                  (2)

Si la primera solución (1) se evapora a sequedad, se obtiene hidróxido de sodio sólido, NaOH,
en lugar de sodio sólido metálico. Esto junto con los productos de burbujas de hidrógeno,
indica que se efectúa una reacción con el disolvente. La evaporación de la solución de cloruro
de sodio (2) permite obtener en NaCl original. La facilidad del proceso de disolución depende
de dos factores: a) el cambio de energía (reacción exotérmica o endotérmica) y b) el cambio


     COMPENDIOS DE QUÍMICA GENERAL PARA ESTUDIANTES DE CIENCIAS E INGENIERÍA
SOLUCIONES Y PROPIEDADES COLIGATIVAS                        63
               3
de desorden (entropía) que acompaña al proceso, es decir el proceso de disolución procede
hacia la disminución de la energía del sistema, lo cual corresponde a un proceso exotérmico y
hacia un incremento de desorden del sistema.

El primer factor que se refiere al cambio de energía se denomina calor de solución, ∆Hsolución.
En un líquido puro, las fuerzas intermoleculares se producen todas entre moléculas similares;
al mezclar un líquido con un sólido, cada molécula experimenta fuerza procedente de otras
moléculas o iones diferentes y también de moléculas similares. Las fuerzas relativas de estas
interacciones ayudan a determinar el grado de solubilidad de un soluto en un disolvente. Las
principales interacciones que afectan la disolución de un soluto en disolvente son:
                          1.    Atracciones soluto-soluto

                          2.    Atracciones disolvente-disolvente

                          3.    Atracciones disolvente-soluto

La disolución se favorece cuando el valor de los dos primeros incisos son relativamente
pequeños y el del tercero es relativamente grande. Es preciso vencer las atracciones
intermoleculares o interiónicas entre las partículas de soluto para que se disuelva, esta parte
del proceso requiere consumo de energía. La separación de moléculas de disolvente también
consume energía. Sin embargo, cuando las partículas de soluto y las moléculas del disolvente
interaccionan en la solución se libera energía y el proceso de disolución es exotérmico.

Muchos sólidos se disuelven en líquidos por procesos 4endotérmicos. La razón de que estos
procesos se produzcan es que la endotermicidad es contrarrestada por un mayor incremento en
el desorden del soluto que acompaña al proceso de disolución. Casi todos los procesos de
disolución están acompañados de un incremento de desorden tanto en el disolvente como en
el soluto. Por tanto, este factor de desorden suele ser favorable a la solubilidad.

3.6 UNIDADES DE CONCENTRACIÓN


L   a concentración de un soluto es la cantidad de soluto disuelto en una cantidad dada de
    disolvente o de solución. La cantidad de disolvente o de solución se puede expresar en
términos de volumen o en términos de masa o de cantidad molar. Así, hay varias formas para
expresar la concentración de una solución.

La concentración de una solución suele expresarse como masa de soluto por unidad de
volumen; pero es mas significativa expresarla en términos de número de moles por unidad de
volumen. Los términos de concentración mas importantes utilizados por los químicos son:
molaridad, molalidad, normalidad y fracción molar.
3.6.1 Molaridad




3
 Whitten Kennet W. define entropía como la medición del desorden de un sistema, mientras
mayor sea el desorden de un sistema, mayor será su entropía.
4
 Proceso endotérmico es cuando en el proceso de disolución de un soluto, en el disolvente, éste absorbe energía
del medio ambiente.

    COMPENDIOS DE QUÍMICA GENERAL PARA ESTUDIANTES DE CIENCIAS E INGENIERÍA
64                             SOLUCIONES Y PROPIEDADES COLIGATIVAS

L    a concentración molar , o molaridad [M], se define como los moles de soluto disueltos
     en un litro de solución.
                                                              moles de soluto
                                         Molaridad [ M ] =                                               (3.1)
                                                             litro de solución


Una solución acuosa de HCl que es 0.2 M, contiene 0.2 moles de HCl por litro de solución, en
la práctica si se quiere preparar esta solución, se agrega a un matraz volumétrico aforado de
1.00  una cierta cantidad de agua por ejemplo ¼ , al cual añadimos 0.2 moles de HCl ( debe
hacerse cálculos a partir de los datos que lleva la etiqueta del frasco). En seguida se llena el
matraz con agua adicional a la marca hasta el cuello y se mezcla la solución.


Ejemplo 3.2.- 0.25 g de una muestra de sulfato de cobre pentahidratado (CuSO 4∗5H2O) se
coloca en un matraz aforado de 250 cc. El matraz se llena con agua hasta la marca en el
cuello. ¿Cuál es la molaridad de la solución resultante?

Solución.- Para determinar la molaridad, se necesitan los moles de soluto.
Por consiguiente convertiremos los gramos de CuSO4.5H2O a moles:

                               1 mol CuSO 4 ∗ 5H 2 O
n = 0.25 g CuSO 4 ∗ 5H 2 O ∗                         = 0.1 mol CuSO 4 ∗ 5H 2 O
                                     249.54 g

El volumen de la solución es 0.25 litros, en consecuencia la molaridad es:

                                  0.1 mol CuSO 4 ∗ 5H 2 O
                  Molaridad =                             = 0.4 M
                                        0.25 litros

3.6.2 Equivalente - gramo de ácidos y bases


S   e define un equivalente – gramo de un ácido como el peso en gramos de éste que puede
    producir un mol de H+ y un equivalente – gramo de una base como el peso en gramos de
ésta que pueda producir un mol de OH-. El peso equivalente o equivalente - gramo de un
ácido se obtiene dividiendo su peso fórmula o peso molecular entre el número de hidrógenos
ácidos que aporta una molécula del ácido. El peso equivalente o equivalente - gramo de una
base se obtiene dividiendo su peso fórmula o peso molecular entre el número de iones
hidróxido. A partir de definiciones anteriores, se ve que un equivalente gramo de cualquier
ácido reacciona con un equivalente – gramo de cualquier base.

No es cierto que un mol de cualquier ácido reacciona con un mol de cualquier base en una
reacción química específica. En consecuencia, según la definición de equivalentes, 1 eq-g
ácido = 1 eq-g base. Por lo general, se puede escribir la siguiente expresión para todas las
reacciones ácido base que llegan hasta el 5punto final.

                      Eq de ácido = eq de base o meq de ácido = meq de base                         (3.2)
                   Donde: meq = miliequivalentes, 1 eq = 1000 meq

3.6.3 Equivalente – gramo en reacciones Redox
5
 Punto final es el punto en el cual el indicador cambia de color y se define la titulación en una reacción de
neutralización.
  COMPENDIOS DE QUÍMICA GENERAL PARA ESTUDIANTES DE CIENCIAS E INGENIERÍA
SOLUCIONES Y PROPIEDADES COLIGATIVAS                             65


E    n reacciones de oxidación – reducción se debe recordar que un agente oxidante acepta
     electrones, y un agente reductor los produce. El principio de equivalente permite hacer
cálculos estequiométricos en reacciones redox. El equivalente gramo del agente oxidante y del
agente reductor es respectivamente:
                                                               peso molecular
                               Eq − g (agente oxidante) =
                                                               No. e − ganados

                                                              peso molecular
                               Eq − g (agente reductor) =
                                                              No. e − perdidos
3.6.4 Normalidad


L   a concentración normal , o normalidad [N], de una solución se define como el número
    de pesos equivalentes o simplemente equivalentes (eq) de soluto por litro de solución.
                                    Número de peso equivalente de soluto
                  Normalidad [ N ] =                                     o
                                               litro de solución
                                    No. de miliequivalente de soluto
                  Normalidad[ N ] =
                                           litro de solución


                                                                                        (3.3)


Ejemplo 3.3 ¿Cuál es la normalidad de una solución que contiene 10 g de H 2SO4 en 500 ml
de solución?

Solución.- El número de moles de H2SO4 es:

                                                1 mol H 2SO 4
                           n = 10 g H 2SO 4 ∗                 = 0.102 mol H 2SO 4
                                                 98 g H 2SO 4


El volumen de la solución es 0.5 litros, en consecuencia la normalidad es:

                                       0.102 moles H 2 SO 4 2eq − gH 2 SO 4
                     Normalidad =                          ∗                = 0.408 N
                                            0.5 litros       1molH 2 SO 4




3.6.5 Molalidad


L   a concentración molal , o molalidad [m], se define como los moles de soluto disueltos
    en un kilogramo de disolvente.
                                                              moles de soluto
                                           Molalidad [ m] =                             (3.4)
                                                              kg de disolvente




  COMPENDIOS DE QUÍMICA GENERAL PARA ESTUDIANTES DE CIENCIAS E INGENIERÍA
66                     SOLUCIONES Y PROPIEDADES COLIGATIVAS
Ejemplo 3.4.- El tolueno, C6H5CH3, es un compuesto líquido similar al benceno, C6H6. Es la
materia prima para otras sustancias, entre ellas el trinitrotolueno (TNT). Encuentre la
molalidad del tolueno en una solución que contiene 5 g de tolueno en 25 g de benceno.

Solución.- El número de moles de tolueno es:
                                            1 mol de C 6 H 5 CH 3
                       5 g C 6 H 5 CH 3 ∗                         = 0.0543 mol C 6 H 5 CH 3
                                              92 g C 6 H 5 CH 3

La masa del benceno que constituye el disolvente es: 0.025 kg C6H6

                               0.0543 mol C 6 H 5 CH 3         mol soluto
                 molalidad =                           = 2.172             = 2.172 molal
                                   0.025 kg C 6 H 6            kg solvente



3.6.6 Formalidad


L    a formalidad de una solución es el número de peso fórmula en gramos de soluto por litro
     de solución; el símbolo de esta unidad es F. La formalidad es muy similar a la molaridad,
por ejemplo 1 formal que se simboliza 1F del ácido clorhídrico es igual a su peso fórmula, es
decir 36.5 g HCl, por litro de disolución.

3.6.6 Fracción molar


L   a fracción molar de una sustancia A (χA), componente de una solución se define como
    los moles de sustancia A divididos entre los moles totales de solución, esto es:
                                     moles de sustancia A       n
                            χA =                               = A                            (3.5)
                                   moles totales de la solución n t
3.7 DILUCIÓN


E   n la práctica de laboratorio con frecuencia se usan soluciones concentradas de
    concentración conocida para preparar a partir de ellas soluciones diluidas. Por tanto, es
importante ver cómo los cambios de volumen afectan la concentración de una solución.

Cuando la concentración se expresa en una escala volumétrica, la cantidad de soluto
contenido en un volumen determinado de la solución es igual al producto del volumen por la
concentración, es decir:
                         Cantidad de soluto disuelto = volumen ∗ concentración

Cuando se diluye una solución, el volumen aumenta y la concentración disminuye, pero la
cantidad total de soluto permanece constante. Por esta razón, dos soluciones de
concentraciones diferentes pero que contengan las mismas cantidades de soluto, están
relacionadas por la siguiente expresión:

                       Volumen1 ∗ Concentración1 = Volumen2 ∗ Concentración2

                                                        V1 ∗ C1 = V2 ∗ C2                     (3.6)

Para reacciones de neutralización ácido – base la ecuación correspondiente y de mucha
utilidad es:
  COMPENDIOS DE QUÍMICA GENERAL PARA ESTUDIANTES DE CIENCIAS E INGENIERÍA
SOLUCIONES Y PROPIEDADES COLIGATIVAS                              67
                                        N1∗V1 = N2∗V2                                     (3.7)

Donde N1 y N2 son concentraciones normales y V1 y V2 son los volúmenes requeridos para la
neutralización de una base y una ácido. No obstante se usa también para diluciones.

Ejemplo 3.4.- 0.25  de una solución contiene 75 g de NaCl por litro. a) ¿A qué volumen se
debe diluir para obtener una solución de concentración 15 g de NaCl por litro?, b) ¿qué
cantidad de agua se necesita para este propósito?

Solución: a) Considerando la expresión (3.6) y despejando Volumen se tiene:

                                            V1 ∗ C1 = V2 ∗ C2

                                   V ∗C   0.25  ∗ 75 g/
                               V2 = 1 1 =                 = 1.25 
                                    C2       15 g/

b) La cantidad de agua que se requiere para esta dilución es:

                                           (1.25 – 0.25)  = 1 

Ejemplo 3.5.- Calcule el volumen aproximado del agua que se debe agregar a 250 cm 3 de una
solución 1.25 N para preparar una solución de concentración 0.5 N (desprecie los cambios en
volumen)

Solución: Considerando la expresión (3.7) se puede determinar inicialmente el volumen de la
nueva concentración y luego el volumen de agua requerido en la dilución de la concentración
original.
                                             N1∗V1 = N2∗V2

                                     N1V1 1.25 N ∗ 250 cm 3
                              V2 =       =                  = 625 cm 3
                                      N2        0.5 N
La cantidad de agua es:
                                      (625 – 250) cm3 = 375 cm3

Ejemplo 3.6.- Si una solución se prepara a partir de 1 mol de etilenglicol y 9 mol de agua, los
moles totales de la solución son 10 moles, en consecuencia la fracción molar del etilenglicol
es:
                                           1 mol etilenglicol
                                      χ=                      = 0.1
                                               10 moles

Recordemos que la suma de las fracciones de todos los componentes de una solución es igual
a 1.
                                          n
                                  (3.8)     X =1∑
                                                i =1
                                                       i
                                                                   Algunas estrategias:
3.8 REACCIONES QUÍMICAS EN
SOLUCIONES                                         La molaridad y la normalidad se calculan
                                                   partiendo de la densidad y del porcentaje


C    omo es de conocimiento, se discutió el método de composición de launa solución. En
                                                   soluciones diluidas      densidad de la
     para resolver problemas en que intervienen solución es muy semejante a la densidad
reacciones químicas, ahora estamos preparados para del disolvente puro. Para calcular la
                                                   molalidad y fracción molar solo es
  COMPENDIOS DE QUÍMICA GENERAL PARA ESTUDIANTES DE conocer el E INGENIERÍA
                                                   necesario CIENCIAS porcentaje de
                                                   composición de la solución.
68                     SOLUCIONES Y PROPIEDADES COLIGATIVAS
extender el método a reacciones en solución y poder efectuar cálculos estequiométricos a
partir de un volumen de una concentración determinada a la cantidad de soluto presente.

Los problemas en esta reacción se basan en dos principios:

          El número de equivalentes de todas las especies en una reacción química es el mismo.
          Volumen ∗ normalidad = número de equivalentes

Ejemplo 3.7 ¿Cuántos mililitros de NaOH 4.0 N se necesitan para neutralizar 20 ml de HCl
3.0 N?

Solución: Se puede usar el concepto de miliequivalentes, esto es:
                                              4.0 meq           3.0 meq
                                    4.0 N =        3
                                                      y 3.0 N =
                                                cm                cm 3
Si:       N1 = 4.0 N
          N2 = 3.0 N
          V2 = 20 ml HCl
          V1 = ¿?
                                          N 2 V2 3.0 ∗ 20 ml
                                  V1 =          =            = 15 ml NaOH
                                           N1        4.0

Ejemplo 3.8.- ¿Cuántos gramos de NaOH se necesitan para neutralizar 90 ml de HCl 1.5 M?

Solución: Para resolver el problema es conveniente transformar la concentración molar en
normalidad, esto es,
                                            1 eq-g HCl = 1 mol HCl
En consecuencia:
                                          moles HCl 1 eq − g HCl
                                    1.5            ∗             = 1.5 N
                                                    1 mol HCl

Por tanto, en una reacción ácido base se tiene:
                                            No eq HCl = No eq NaOH
                                         1.5 meq HCl
                      90 ml solución ∗                 = 135 meq HCl = 135 meq NaOH
                                         1 ml solución

Por consiguiente la masa de NaOH requerida es:
                                          1 eq − g NaOH   40 g NaOH
                      135 meq NaOH ∗                    ∗            = 5.4 g NaOH
                                         1000 meq NaOH 1 eq − g NaOH

Ejemplo 3.9.-Encuentre el peso equivalente de KMnO4 en la siguiente reacción:

                           MnSO4 + KMnO4 + H2O ⇔ MnO2 + H2SO4 + K2SO4

b) ¿Cuántos gramos de MnSO4 se oxidan con 50 cm3 de una solución de KMnO4 0.1 N?

Solución: Para resolver el problema efectuaremos la igualación de la ecuación química.

             [Mn+2 + SO4= ] + [K+ + MnO4- ] + H2O0 ⇒ MnO20 + [2H+ + SO4= ] + [2K+ + SO4=]

                 2∗          3 e- + 4 H+ + MnO4- ⇒ MnO20 + 2 H2O0
  COMPENDIOS DE QUÍMICA GENERAL PARA ESTUDIANTES DE CIENCIAS E INGENIERÍA
SOLUCIONES Y PROPIEDADES COLIGATIVAS                                            69

                             3∗           2 H2O0 + Mn+2          ⇒ MnO20 + 4 H+ + 2 e-
                                     2 MnO + 2 H2O + Mn+2
                                          4
                                           -
                                                                 ⇒ 5 MnO20 + 4 H+

         La ecuación igualada es:

                                  3 MnSO4 + 2 KMnO4 + 2 H2O ⇔ 5 MnO2 + 2 H2SO4 + K2SO4

         a) el peso equivalente del permanganato de potasio es:
                                                                  158 g
                                                eq − g KMnO 4 =         = 52.67 g
                                                                    3
         b) La masa de sulfato manganoso requerido es:
                             0.1 eq KMnO 4 1 mol KMnO 4 3 mol MnSO 4 151 g MnSO 4
                 50 cm 3 ∗                ∗            ∗            ∗             = 0.3775 g MnSO 4
                               1000 cm 3    3 eq KMnO 4 2 mol KMnO 4 1 mol MnSO 4

         3.9 PROPIEDADES COLIGATIVAS


         A    lgunas propiedades físicas de las soluciones difieren mucho de las del disolvente puro.
              Por ejemplo, el agua pura se congela a 0 ºC, pero las soluciones acuosas se congelan a
         temperaturas menores. El etilenglicol se adiciona al agua de los radiadores de los automóviles,
         pues es un anticongelante ya que abate (disminuye) el punto de congelación de la solución;
         también eleva el punto de ebullición de la solución sobre la del agua pura, permitiendo que el
         motor funcione a una temperatura mas alta.

         Una solución, que conste de dos o mas componentes, carece de las propiedades físicas
         constantes de una sustancia pura; estas propiedades dependen de la concentración de las
         partículas del soluto y no de su naturaleza. Tales propiedades se conocen como propiedades
         coligativas y éstas son: el descenso de la presión de vapor; la depresión en el punto de
         congelación; la elevación en el punto de ebullición y la presión osmótica.

         Las propiedades coligativas se pueden usar en la determinación de los pesos moleculares de
         las sustancias disueltas y pueden dar además información acerca de las propiedades del soluto
         si se conocen las propiedades del disolvente. Todas las soluciones obedecen las leyes que se
         discuten en este capítulo , cuando la concentración es suficientemente baja. Por esta razón se
         les conoce como leyes de las soluciones diluidas.

         3.9.1 Descenso de la presión de vapor y la ley de Rault


         M      uchos experimentos han demostrado que las soluciones que contienen líquidos no
                volátiles o sólidos como solutos, siempre tienen presiones de vapor mas bajas que los
         disolventes puros.




Fig. A
         FIGURA 3.6
         La figura A muestra la presión de vapor del agua pura, la figura B el equilibrio de dos sistemas que contienen agua
                                                   Fig. B
         pura, en la figua C la presión manométrica (diferencia de altura) de los dos líquidos se debe al vapor de agua
                                                                                                Fig. C
         pura. Esta presión es menor sobre la solución azucarada, por que hay menos moléculas de agua por unidad de área
         de COMPENDIOS DE QUÍMICA GENERAL PARA ESTUDIANTES DE CIENCIAS E INGENIERÍA
            superficie al evaporarse.
70                       SOLUCIONES Y PROPIEDADES COLIGATIVAS




Cuando un soluto no volátil se disuelve en un líquido; parte del volumen total de la solución
es ocupada por moléculas de soluto, y por lo tanto, hay menos moléculas de disolvente por
unidad de área en la superficie. Esto ocasiona el descenso de presión de vapor del disolvente.
El descenso de la presión de vapor asociados con solutos no volátiles y no ionizables se
resume en la ley de Rault:
              “La presión de vapor de un disolvente en una solución es igual a la presión de vapor del
              disolvente puro multiplicada por la fracción molar del mismo en la disolución”.
Matemáticamente se expresa:
                                                P = Xd.Pº                                        (4.9)

Donde: Xd, representa la fracción molar del disolvente en la solución, Pº es la presión de
vapor del disolvente puro y P es la presión de vapor del disolvente en la solución. Esta ley
permite calcular pesos moleculares.

Ejemplo 3.10.- La presión de vapor del agua pura a 25 ºC es 23.76 mmHg y la presión de
vapor de una mezcla formada por 5.4 g de soluto no volátil en 90 g de agua es 23.32 mmHg.
Determinar el peso molecular de dicho soluto.

Solución: De acuerdo a la ley de Rault:
                                                 P = Xd.Pº

                                                   P        23.32
                                            Xd =        =         = 0.9815
                                                   Po       23.76
Puesto que la fracción molar se define como:

                                          moles de sustancia A       n
                                 XA =                               = A
                                        moles totales de la solución n t

Se puede expresar en términos de sus pesos moleculares:
                                                            md
                                                            Md
                                           0.9815 =
                                                        md  m
                                                           + s
                                                        Md Ms
reemplazando datos:
                                                       90 g
                                          0.9815 =      18
                                                   90 g 5.4 g
                                                        +
                                                    18     Ms
despejando Ms:
                                Ms= 57.3 g/mol

Ejemplo 3.11.- Determine la presión de vapor a 25 ºC de una solución acuosa que consta de
10 g de sacarosa, C12H22O11 y 75 g de H2O.

Solución: De acuerdo a la ley de Rault:
  COMPENDIOS DE QUÍMICA GENERAL PARA ESTUDIANTES DE CIENCIAS E INGENIERÍA
SOLUCIONES Y PROPIEDADES COLIGATIVAS                                               71
                                       P = Xd.Pº
La fracción molar del disolvente es:
                                                              md
                                                              Md
                                                    Xd =
                                                           md  m
                                                              + s
                                                           Md Ms

                                                        4.17
                                             χd =                 = 0.993
Por consiguiente La presión de                      4.17 + 0.0292            vapor es:      Pv = 0.993 ∗ 23.76
mmHg = 243.59 mmHg

3.9.2 La ley de Rault y las soluciones de solutos volátiles


E   n soluciones en las cuales tanto el disolvente como el soluto tienen una presión de vapor
    apreciable, se puede aplicar la ley de Rault a ambos componentes:

Para lograr cierta comprensión de tales mezclas, considere una solución ideal que contiene
dos componentes, A y B, según la ley de Rault:
              PA = XA.PAº                     y                    PB = XB.PBº

La presión total del sistema es entonces, la suma de las presiones parciales de cada
componente volátil:
                                 Ptotal = PA + PB = XA.PAº + XB.PBº            (3.10)

Ejemplo 3.12.- ¿Cuál es la presión de vapor que ejerce una mezcla formada por 100 gramos
de benceno y 100 g de tolueno a 25 ºC? A dicha temperatura las presiones del vapor de
benceno y tolueno puros son, respectivamente 95.1 y 28.4 mmHg.

Solución: De acuerdo a la expresión (3.6)
                                                  Ptotal = PC6H6 + PC7H8

                                        P total = XC6H6* PC6H6º + XC7H8 * PC7H8º
Las fracciones molares de las dos sustancias son:
                                       100 g                               100 g
                       n    benceno =           = 1.282       n tolueno =         = 1.087
                                           g                                   g
                                      78                                  92
                                          mol                                 mol
                                        n total = 1.282 + 1.087 = 2.369
                                    1.282                                  1.087
                       χ C6 H 6   =         = 0.541              χ C7 H8 =        = 0.459
                                    2.369                                   2.369




La presión de vapor será:
                              Pt = 0.549* 95.1 mmHg + 0.459 * 28.4 mmHg

                                         Pt = 52.21 + 13.04 = 65.25 mmHg

  COMPENDIOS DE QUÍMICA GENERAL PARA ESTUDIANTES DE CIENCIAS E INGENIERÍA
72                     SOLUCIONES Y PROPIEDADES COLIGATIVAS
Las soluciones pueden representarse gráficamente. La figura 3.7 muestra la ley de Rault para
una solución ideal de un soluto en
un líquido volátil. La presión de
vapor que ejerce el líquido es




                                            PRESION DE VAPOR




                                                                                              PRESION DE VAPOR
                                            DEL DISOLVENTE




                                                                                              DEL DISOLVENTE
proporcional a su fracción molar
en la solución.

Este diagrama se cumple para:
               P = Xd.Pº
                                                                                                                          III
La figura 3.8 muestra la ley de
Rault aplicado a soluciones que                                                                                      II
tienen dos componentes volátiles.
Este diagrama se cumple para:
                                                                                                                 I
   Ptotal = PA + PB = XA.PAº + XB.PBº

En la figura 3.8 la línea (I) es la            A Xdisolvente     B        A Xdisolvente        B
presión parcial de A y la línea (II)                 Figura 3.7                  Figura 3.8
es la presión parcial de B y la
línea (III) es la presión total para diferentes concentraciones de los dos líquidos volátiles.

La figura 3.9 muestra una desviación negativa de la ley de Rault, Las fuerzas intermoleculares
existentes en la solución son superiores a las fuerzas intermoleculares de los componentes
puros aisladamente.

La figura 3.10                                                                                                   muestra la presión
de vapor superior                                                                                                a      la    presión
                                                           PRESION DE VAPOR
                                                           DEL DISOLVENTE




predicha por la                                                                                                  ley      de    Rault
(desviación                                                                                                      positiva).       Las
fuerzas                                                                                                          intermoleculares
existentes en la                                                                                                 solución son más
débiles que las de                                                                                               los     componentes
puros




                        A   Xdisolvente       B                        A      Xdisolvente      B
                               Figura 3.9                                       Figura 3.10
independientemente.

3.9.3 LEY DE HENRY


E    l efecto de la presión sobre la solubilidad de un gas en un líquido se puede predecir de
     manera cuantitativa. Para soluciones diluidas de un gas no reactivo en un líquido, puede
  COMPENDIOS DE QUÍMICA GENERAL PARA ESTUDIANTES DE CIENCIAS E INGENIERÍA
SOLUCIONES Y PROPIEDADES COLIGATIVAS                       73
aplicarse una expresión muy similar a la ley de Rault, conocida como la ley de Henry, cuya
expresión matemática es:
                                   Pgas = K∗Xgas                                    (3.11)

Donde Pgas es la presión del gas sobre la superficie de la solución y k es una constante para
un gas y un disolvente determinados a una determinada dada. Xgas representa la fracción
molar del gas disuelto; la relación es válida a bajas concentraciones y a bajas presiones.

Ejemplo 3.13.- Si 29 mg de N2 se disuelven en un litro de agua a 0 ºC y 760 mmHg de
presión de N2, ¿qué masa de N2 se disolverá en un litro de agua a 0 ºC y 5 atm de presión?

Solución: De acuerdo con la ecuación (3.11), en principio se determinará la constante de
Henry K, para luego determinar la masa de nitrógeno disuelto en las nuevas condiciones de
presión y concentración.

La fracción molar del gas es:
                                             1g N2     1 mol N 2
                             29 mg N 2 ∗             ∗           = 1.036 ∗ 10 −3 mol N 2
                                           1000 g N 2 28 g N 2

                                          1000 ml 1 g 1 mol H 2 O
                             1 H 2 O ∗           ∗    ∗           = 55.56 mol H 2 O
                                          1 H 2 O 1 ml 18 g H 2 O

                                             1.036 ∗ 10 −3 molN 2
                                    χ N2 =                        = 1.8646 ∗ 10 −5
                                                   55.561



La constante de Henry será:
                                            Pgas    1 atm
                                     K=          =             = 5.363 ∗10 4
                                            χgas 1.8646 ∗10 −5

La masa de nitrógeno disuelta en 1 litro de agua a 5 atm será:
                                               Pgas      5atm
                                     χ N2 =         =           4
                                                                  = 9.323 ∗10 −5
                                                K     5.363 ∗10
1 litro de H2O = 55.56 mol
                                              nN 2                 nN2
                                χ N2 =                     =                   = 9.323 ∗10 −5
                                          n N 2 + n H 2O       n N 2 + 55.56


Resolviendo la ecuación se tiene:
                                                  28 g N 2 1000 mg N 2
                        5.18 ∗ 10 −3 mol N 2 ∗             ∗           = 145.04 mg N 2
                                                 1 mol N 2   1 g N2


3.9.4 Aumento del punto de ebullición


R    ecordemos que el punto de ebullición de un líquido es la temperatura a la cual la presión
     de vapor se iguala a la presión aplicada en su superficie, por ejemplo la temperatura de
ebullición normal del agua sobre el nivel del mar es 100 ºC

  COMPENDIOS DE QUÍMICA GENERAL PARA ESTUDIANTES DE CIENCIAS E INGENIERÍA
74                      SOLUCIONES Y PROPIEDADES COLIGATIVAS
                   Se ha visto que la presión de vapor de un disolvente a una temperatura dada, desciende por la
                   presencia de un soluto no volátil. Las soluciones de este tipo deben calentarse a temperaturas
                   mas altas que el disolvente puro para que su presión de vapor iguale a la presión atmosférica.

                   El incremento en el punto de ebullición, ∆Te ( en relación al punto de ebullición del
                   disolvente puro), es directamente proporcional al número de partículas de soluto por mol de
                   disolvente. Sabemos que la molalidad expresa el número fijo de moles de disolvente. Así Te
PRESION DE VAPOR




                   es proporcional a la molalidad, como se muestra en la siguiente expresión matemática:
DEL DISOLVENTE




                                                                         ∆Te = Kem                            (3.12)

                                                             ∆Te = Tf(solución) – Tf(disolvente)

                   Ejemplo 3.14.- Una disolución de glicocola preparada al disolver 1.62 g de sustancia en
                   19.62 g de agua, hierve a 100.6 ºC. Hallar el peso molecular de la glicocola. (Ke = 0.52
                   ºC/molal).

                   Solución: Datos:
                   Te = 100.6 ºC
                   Ke = 0.52 ºC/molal

                   De acuerdo a la relación (2.7)                        ∆Te = Kem

                   Se puede determinar la molalidad

                        ΔTe (100.6 − 100)°C
                   m=       =                = 1.154 molal
                         ke   0.52 °C/ molal

                   A partir de este dato se puede evaluar el número de moles de soluto:

                                                 moles de soluto
                                         1.154                   ∗ 19.62 g de agua = 0.0226 moles de soluto
                                                 1000 g de agua

                   En consecuencia el peso molecular de soluto será:

                                                            ms     1.62 g
                                                       M=      =             = 71.68 g/mol
                                                            n s 0.0226 moles

                   Para hallar el peso molecular se puede considerar un segundo método propuesto por el Prof.
                   José Ibarz Aznárez, el cuál expresa:

                   Si una disolución está constituida por a gramos de soluto y A g de disolvente, y el peso
                   molecular del soluto es M, la molalidad de disolución es:

                                                                                a • 1000
                                                                          m=                                  (3.13)
                                                                                 A•M

                   Considerando la expresión (3.8), y despejando M se tiene:



                     COMPENDIOS DE QUÍMICA GENERAL PARA ESTUDIANTES DE CIENCIAS E INGENIERÍA
SOLUCIONES Y PROPIEDADES COLIGATIVAS                                                          75
                                               a • 1000    1.62 • 1000
                                        M=              =               = 71.55 g/mol
                                                A•m       19.62 • 1.154

3.9.5 Descenso del punto de congelación


E     n contraste con el punto de ebullición, el punto de congelación de una solución
      generalmente es mas bajo que el punto de congelación del solvente puro, como muestra
la figura 3.6. La diferencia entre estas dos temperaturas se conoce como depresión en el punto
de solidificación y se designa por ∆Tc, y es proporcional a la concentración molal del soluto.
Esta proporcionalidad, convertida en igualdad se puede expresar por medio de la siguiente
ecuación:

                  Tc = Kcm                         (3.14)

             ∆Tc = Tf(disolvente) – Tf(solución)




                                                                        Presión de vapor [mmHg]
La figura 3.11 muestra como un soluto no
volátil abate la presión de vapor de un
disolvente, el punto de ebullición se eleva y
el punto de congelación desciende con
respecto a los puntos correspondientes en los
disolventes puros. La magnitud de elevación
del punto de ebullición ∆Te es menor que la
magnitud del abatimiento del punto de                                                                                    ∆P
congelación ∆Tc.



Ejemplo 3.15.- Se funde una mezcla de
0.436 g de acetanilida con 14.06 g de
alcanfor de punto de fusión 176.2 ºC. La
mezcla se deja solidificar y enfriar, se reduce                                                   ∆Tc Temperatura [ºC]
                                                                                                                  ∆Te
a polvo, se mezcla íntimamente y se
                                                                 Figura 3.11
calienta. En el momento en que se termina                        Diagrama de fase del H2O y de una solución acuosa
de fundir su temperatura es de 167.0 ºC.
Hallar el peso molecular de la acetanilida.
(kcalcanfor = 40.0 ºC/molal)

Solución: Los datos son los siguientes:
Tc = 176.2 ºC
kc = 40.0 ºC/molal

De acuerdo a la relación (3.7)
                                                            ∆Tc = kcm

Se puede determinar la molalidad:

                                               ΔTc (176.2 − 167.0)°C
                                         m=        =                 = 0.23 molal
                                                kc   40.0 °C/ molal

  COMPENDIOS DE QUÍMICA GENERAL PARA ESTUDIANTES DE CIENCIAS E INGENIERÍA
76                       SOLUCIONES Y PROPIEDADES COLIGATIVAS

A partir de este dato se puede evaluar el número de moles de soluto:

                                       moles de soluto
                             n = 0.23                   ∗ 14.06 g de alcanfor
                                     1000 g de alcanfor
                             n = 0.00323 moles de soluto

En consecuencia el peso molecular de soluto será:

                    ms     0.436 g
               M=      =              = 134.98 g/mol
                    n s 0.00323 moles

Para hallar el peso molecular se puede considerar la expresión (3.8), y despejando M se tiene:

                                    a • 1000 1.62 • 1000
                               M=           =            = 71.55 g/mol
                                     A • m 19.62 • 1.154

              TABLA 3.2
              Algunas propiedades de disolventes comunes
                              PUNTO DE          ke       PUNTO          kc
            SOLVENTE          EBULLICIO       [ºC/m]    FUSION        [ºC/m]
                                N [ºC]                     [ºC]
             H2O               100.00          0.52       0.0         1.86
             C6H6               80.10          2.53       5.50        5.12
             CCl4               76.80          5.02     -22.3        29.80
             C2H5OH             78.40          1.22    -114.6         1.99
             ClCHO              61.20          3.63    -63.50         4.68

3.9.6 Aplicaciones del aumento y descenso del punto de ebullición y fusión
respectivamente

Ejemplo 3.16.- El punto de ebullición de una solución de 0.402 g de naftaleno, en 26.6 g de
cloroformo, es 0.455 ºC mas alto que el del cloroformo puro. ¿Cuál es la constante
ebulloscópica del cloroformo?

Solución: A partir de la expresión (2.6)         ∆Te = Kem, se puede despejar Ke, puesto que ∆Te
= 0.455° y la molalidad es:

                         0.402 g C10 H 8 1000 g CHCl3 1 mol C10 H 8
                    m=                  ∗             ∗               m = 0.118 molal
                         26.6 g CHCl3     1 kg CHCl 3   128 g C10 H 8
                                         ΔTe 0.455 °C
                                  Ke =      =         = 3.86 °C/molal
                                          m   0.118 m

Ejemplo 3.17.- La presión de vapor de una solución acuosa diluida es 23.45 torr a 25 ºC,
mientras que la presión de vapor del agua pura a la misma temperatura es 23.76 torr.
Calcúlese la concentración molal del soluto, y utilícense los valores tabulados de Ke del agua
para predecir el punto de ebullición de la solución.

Solución: Puesto que la concentración molal se define como:


  COMPENDIOS DE QUÍMICA GENERAL PARA ESTUDIANTES DE CIENCIAS E INGENIERÍA
SOLUCIONES Y PROPIEDADES COLIGATIVAS                             77
                                                            moles de soluto
                                   Molalidad [ m] =
                                                            kg de disolvente

se puede considerar la masa del disolvente como 1 kg = 1000 g H2O.

De acuerdo a la ley de Rault:
                                                   P = Xd.Pº

                                               P           23.45
                                        Xd =       o
                                                       =         = 0.987
                                               P           23.76

Puesto que la fracción molar se define como:
                                          moles de sustancia A       n
                                 XA =                               = A
                                        moles totales de la solución n t

Se puede expresar en términos de sus pesos moleculares:
                                                              md
                                                              Md
                                           0.987 =
                                                           md  m
                                                              + s
                                                           Md Ms


reemplazando datos:
                                                     1000 g
                                           0.987 =     18
                                                   1000 g
                                                          + ns
                                                     18

El número de moles del soluto, despejando de la anterior expresión es en consecuencia:
                                               ns = 0.727 mol
y la molalidad será:
                                      0.727 mol de soluto
                                 m=                       = 0.727 molal
                                         1 kg de agua
El punto de ebullición de la solución es:

De acuerdo a la relación (3.7)
                                                   ∆Te = Kem

                                 ∆Te = 0.52 °C/m ∗ 0.727 m = 0.38 °C

La temperatura de ebullición de la solución es:
                                   Te = (100 + 0.38) °C = 100.38 °C

3.10 OSMOSIS Y PRESIÓN OSMÓTICA


L    a osmosis es el proceso espontáneo por el cual las moléculas del disolvente atraviesan
     una membrana semipermeable de una solución de menor concentración de soluto hacia
una solución con mayor concentración de mayor soluto

  COMPENDIOS DE QUÍMICA GENERAL PARA ESTUDIANTES DE CIENCIAS E INGENIERÍA
78                       SOLUCIONES Y PROPIEDADES COLIGATIVAS
            Para definir la presión osmótica es conveniente considerar el sistema de la figura 2.12. El cual
            muestra un experimento en una cámara de presión osmótica.

                                              Rebalse de la solución                             Figura 3.14
             Solución de azúcar                                              Las moléculas de
                           Agua                          Figura 3.13         azúcar no pueden
                                                                             atravesar la
                                                                             membrana
                                                                                                          H = π = ρgh



                             Membrana         Las moléculas de agua
                  Figura 3.12                 atraviesan la membrana                            Disolvente
                                                                                                   puro
                                                                                                 (Agua)
            Figura 3.12 - 3.14
            Experimento de ósmosis. El agua pasa a través de la membrana a la solución de azúcar en el
            compartimiento del sistema. El flujo de agua cesa cuando el líquido en el embudo ejerce hacia abajo una
            presión suficiente (la presión osmótica).


            a) La figura 3.12 muestra el inicio de la determinación de la presión osmótica, los niveles de
            solución del lado izquierdo y del disolvente del lado derecho son iguales.

            b) Después del inicio del experimento, las moléculas del disolvente tienden a fluir hacia la
            solución, entonces observamos rebalse de la solución como era de esperarse, como muestra la
            figura 3.13.

            c) Para evitar el rebalse instalemos un tubo en la cámara de la disolución; durante cierto
            tiempo de iniciado el experimento, el flujo de la moléculas hacia la solución cesa y el sistema
            alcanza el equilibrio cuando el menisco se ha elevado hasta una determinada altura, como se
Fig. 2.13
            puede observar en la figura 3.14. En estas condiciones de equilibrio, la solución se encuentra
            bajo una presión hidrostática mayor que el disolvente puro. La altura del menisco
            multiplicada por la densidad de la solución y la aceleración de la gravedad, dá la presión
            adicional sobre la solución y ésta es la presión osmótica π.
            Por la medición experimental realizada en soluciones diluidas de concentración conocida, se
            sabe que la relación entre la presión osmótica y la concentración está dada simplemente por:

                                                               π = cRT                                           (3.15)

            Donde c es la concentración de la disolución en moles/litro, R es la constante universal de los
            gases y T es la temperatura absoluta. La presión osmótica es proporcional a la temperatura
            absoluta porque afecta el número de colisiones del disolvente con la membrana por unidad de
            tiempo, también es proporcional a la concentración molar, ya que ésta afecta a la diferencia en
            el número de moléculas del disolvente que chocan contra la membrana de cada lado.

            Ejemplo 3.18.- Estime el peso molecular de una macromolécula biológica si se disuelve una
            muestra de 0.194 gramos en 82.4 ml de benceno y se observa una presión osmótica de 14.20
            torr a 25 °C.

            Solución: De la ecuación (3.15) π = cRT:
                                             π           14.20 mmHg                  −4   mol
                               c=    =                = 7.64 ∗ 10
              COMPENDIOS DE QUÍMICA GENERAL PARA ESTUDIANTES DE CIENCIAS E INGENIERÍA
                                         mmHg − 
                                            RT                                             
                                                  62.4              ∗ 298K
                                                          K − mol
SOLUCIONES Y PROPIEDADES COLIGATIVAS                           79

Considerando que el volumen de la solución es 82.4 ml, se puede determinar el número de
moles de la macromolécula biológica:
                                                    mol
                                       7 ∗ 10 − 4       ∗ 0.0824 = 5.768 ∗ 10 − 5 mol
                                                     
El peso molecular de dicha sustancia biológica es:
                                                m   0.194g               g
                                          M=      =        −5
                                                              = 3363.38
                                                n 5.768x10              mol
3.10.1 Ósmosis Inversa


E     l6 proceso de ósmosis inversa se ha aplicado al problema de la purificación del agua, en
      particular el método se ha utilizado para desalar el agua del océano (esto es para eliminar
lar sales del agua de mar y obtener agua que se pueda beber o que sea industrialmente
utilizable). En la ósmosis normal, el disolvente fluye a través de una membrana de una
solución diluida a una solución mas concentrada, el proceso de ósmosis se puede detener.
Figura 3.15
Sistema de desalación que utiliza la ósmosis inversa.
Cuando el agua del océano se somete a una presión mayor que
su presión osmótica, el agua pura pasa a través de una serie de
membranas y dejan detrás una solución de sal más
concentrada. Si se aplica una presión aún mayor, el proceso
osmótico puede ser invertido, entonces el disolvente fluye de
la solución concentrada (que pueda ser agua del océano), a
través de una membrana, a una solución más diluida (que
pueda ser agua mas o menos pura). En la figura 3.15 se
presenta un sistema que utiliza ósmosis inversa para desalar el
agua del océano.

3.11 Propiedades               Coligativas            de     las
Soluciones Ionicas


P   ara explicar las propiedades coligativas de las soluciones iónicas, debe aceptarse que la
    concentración total de iones, es mas importante que la concentración de una sustancia
iónica. Por ejemplo, la depresión de congelación de una solución 0.100 m de cloruro de sodio
es casi el doble del de una solución 0.100 m de glucosa. Puede explicarse esto diciendo que el
cloruro de sodio se disuelve en agua para formar los iones Na+ y Cl-. Cada unidad fórmula de
NaCl da dos partículas.

Para cada propiedad coligativa de las soluciones iónicas se pueden considerar tres valores: el
valor calculado ∆Tm, suponiendo el compuesto formado por moléculas; el valor real, ∆T,
bastante mayor, encontrado experimentalmente; y el valor ideal ∆Ti, mayor todavía, que
puede también calcularse al suponer el compuesto formado por iones que se comportasen en
la disolución como si fueran partículas neutras..

3.11.1 Factor de van’t Hoff

La relación:
                                                                    ΔT
                                                             i=                           (3.16)
                                                                   ΔTm

6


    COMPENDIOS DE QUÍMICA GENERAL PARA ESTUDIANTES DE CIENCIAS E INGENIERÍA
80                      SOLUCIONES Y PROPIEDADES COLIGATIVAS

donde i es mayor a la unidad, se conoce como factor de van’Hoff. Cuando se suponía que los
electrolitos estaban constituidos por moléculas se aceptó que una fracción de las mismas se
disociaba en iones y que se establecía un equilibrio entre las moléculas sin disociar y los iones
formados. La fracción de las moléculas ionizadas o disociadas se denomina grado de
disociación. Para los electrolitos fuertes se establece actualmente la relación:

                                                 ΔT
                                                     =g                                   (3.17)
                                                 ΔTi

donde g es menor que la unidad y que se conoce como coeficiente osmótico. Cuanto mas se
acerca a la unidad el valor de g mayor es el comportamiento ideal de los iones en la
disolución iónica.

Si una supuesta molécula del electrolito se disocia en ν iones, es evidente que:

                               ∆Ti = ν∆Tm , y por tanto;

                                                      i
                                                 g=                                       (3.18)
                                                      υ

En el caso de los electrolitos débiles, si α es el grado de ionización, y una molécula forma
realmente ν iones, 1 mol del electrolito dará lugar a να moles y quedaran sin ionizar ( 1 - α)
moles, por lo cual, en vez de un mol de compuesto tendremos:

                                             [1+(ν - 1)α]

moles de partículas, y puesto que cualquier propiedad coligativa es i veces mayor que el valor
teórico correspondiente al numero de moléculas o moles disueltos, tendremos

                                       i = 1 + (ν - 1)α                                   (3.19)
y
                                            i −1
                                       α=                                                 (3.20)
                                            υ −1

Esta expresión se aplica también corrientemente a los electrolitos fuertes, aunque ya se ha
indicado que es incorrecto hablar en ellos de grado de disociación. Para no romper con esta
costumbre nos referiremos nosotros a un grado de disociación aparente para explicar el
comportamiento de los electrolitos fuertes.

El factor de van’t Hoff i para soluciones iónicas en las expresiones de las propiedades
coligativas se puede escribir:

Descenso de la presión de vapor
                                                   md
                                       P = P°      Md
                                                md    ms
                                                                                          (3.21)
                                                   +i
                                                Ms    Ms
Aumento del punto de ebullición
                                       ∆Te = iKem                                         (3.22)
Descenso del punto de fusión
    COMPENDIOS DE QUÍMICA GENERAL PARA ESTUDIANTES DE CIENCIAS E INGENIERÍA
SOLUCIONES Y PROPIEDADES COLIGATIVAS                               81
                                          ∆Tc = iKcm                                    (3.23)
Presión osmótica
                                          π = icRT                                      (3.24)

Ejemplo 3.19.- A 100 °C la presión de vapor de una disolución de 10.0 g de nitrato cálcico en
150 g de agua es de 746.8 mmHg. Hallar el grado de disociación aparente del nitrato cálcico
en esta disolución.

Solución: Para hallar el grado de disociación aparente (véase ecuación 3.15) del nitrato
cálcico (electrolito fuerte) debemos determinar el coeficiente de van’t Hoff a partir de la
ecuación (3.16) y el número de iones ν

                                Ca(NO3)2 = [ Ca++ + 2NO3-]            ν=3
Así que, de la ecuación (3.16):
                                                         md
                                             P = P°      Md
                                                      md    ms
                                                         +i
                                                      Ms Ms

reemplazando datos:
                                                          150
                                          746.8 = 760      18
                                                      150     10
                                                           +i
                                                       18     164

efectuando operaciones con una máquina electrónica:

                                                         8.33
                                          0.982 =
                                                    8.33 + 0.0608 i

                                                    i = 2.42
Aplicando la expresión (3.15)
                                             i − 1 2.42 − 1
                                        α=        =         = 0.71
                                             υ −1    3−1

Ejemplo 3.20.- Una disolución 0.065 molal de cloruro cálcico empieza a congelar a – 0.322
°C, hallar: a) el grado de disociación del cloruro cálcico y b) la concentración de los iones
cloruro y de los iones calcio en la disolución. Suponer la densidad de la disolución igual a la
unidad. Kc(H2O = 1.86 °C/mol)

Solución: a) Para hallar el grado de disociación haremos uso de la ecuación (2.15), sin
embargo es preciso evaluar i y ν.

El cloruro de calcio se disocia:
                                    CaCl2 = [ Ca++ + 2Cl- ]       ν=3

De la ecuación (3.18) ∆Tc = iKcm: Despejamos i:
                                         ΔTc     [ 0 − (−0.322) ]
                                   i=          =                  = 2.66
                                        Kc ∗ m     1.86 ∗ 0.065


  COMPENDIOS DE QUÍMICA GENERAL PARA ESTUDIANTES DE CIENCIAS E INGENIERÍA
82                        SOLUCIONES Y PROPIEDADES COLIGATIVAS
Por consiguiente
                                                 2.66 − 1
                                            α=            ∗ 100 = 0.83
                                                  3 −1

b) Para hallar la concentración molar de los iones, determinaremos en primera instancia la
concentración del cloruro de calcio: La masa total de la solución se puede calcular:

                                0.065 mol CaCl 2 111 g CaCl 2 7.215 g CaCl 2
                                                ∗              =
                                  1000 g H 2 O    1 mol CaCl 2   1000 g H 2 O

Es decir la masa de la solución es:
                                                 msolución = 1000 g
puesto que la densidad es 1.0 g/ml

El volumen de la solución es:
                                              V = 1000 ml = 1.000 

La concentración molar de la solución iónica de CaCl2 es:

                                          0.065 mol de CaCl 2
                                    M=                        = 0.065 molar
                                                1.000 
y las concentraciones de los iones determinamos a partir de : 0.065 ∗ 0.83 = 0.0540
                                          CaCl2 =         Ca++ + 2Cl-
                                           1 mol         1 mol 2 mol

[ Ca++ ]= 0.0540 molar, la relación es 1:1

[ Cl- ] = 2 ( 0.0540) = 0.108 molar la relación es 1:2
                                         PROBLEMAS RESUELTOS


3.1 Una solución de ácido sulfúrico tiene densidad de 1.84 g/ml y contiene 98% en masa de
H2SO4. ¿Qué volumen ocuparán 200 g de H2SO4 puro?

Solución: Se puede considerar los conceptos básicos de factores de conversión:
                                100 g de solución 1 cm 3 de solución
              200 g H 2SO 4 ∗                    ∗                    = 110.91 cm 3 de solución
                                  98 g H 2SO 4     1.84 g de solución

                                 110.91cm 3 = VH 2 O + VH 2 SO 4              (1)
El volumen de agua es:
                             100 g de solución      2 g de H 2 O    1 cm 3 H 2 O
           200 g H 2SO 4 ∗                     ∗                  ∗              = 4.082 cm 3 H 2 O
                               98 g H 2SO 4      100 g de solución 1 g H 2 O

En consecuencia, considerando la ecuación (1), el volumen de H2SO4 puro es:

                                  VH 2SO 4 = 110.91 − 4.082 = 106.82cm 3 H 2SO 4


  COMPENDIOS DE QUÍMICA GENERAL PARA ESTUDIANTES DE CIENCIAS E INGENIERÍA
SOLUCIONES Y PROPIEDADES COLIGATIVAS                                                 83

3.2 Calcule la molalidad, fracciones molares de soluto y disolvente, y la molaridad de las
siguientes soluciones: a) una solución acuosa de H2SO4 que es 50 % en masa y tiene una
densidad de 1.4 g/ml, b) una solución acuosa de sacarosa, C12H22O11, que es 19 por ciento
sacarosa en masa, y tiene una densidad de 1.08 g/ml, y c) una solución compuesta de 24.4 g
de NaOH y 97.6 de H2O con un volumen de 100 ml.

Solución: a) La molalidad de la solución de H2SO4 se puede determinar a partir de la
composición centesimal, vale decir:

                             50 g H 2SO 4 1000 g H 2 O 1 mol H 2SO 4
                       m=                ∗            ∗              = m = 10.2 molal
                              50 g H 2O    1 kg H 2 O   98 g H 2SO 4

La fracción molar se determina calculando previamente los moles del soluto y disolvente:

                                                     1 mol H 2SO 4
                                 50 g H 2SO 4 ∗                    = 0.510 mol H 2SO 4
                                                      98 g H 2SO 4
                                                  1 mol H 2 O
                                 50 g H 2 O ∗                 = 2.78 mol H 2O
                                                  18 g H 2 O
La fracción molar será entonces :

                                                                   0.510
                                                      X H 2 SO 4 =       = 0.155
                                                                   3.290
                                                      X H2O   = 1 − 0.155 = 0.845

La molaridad se calcula considerando la densidad de la solución

                       g Solución   50 g H 2SO 4   1 mol H 2SO 4 1000 mlSol.
             M = 1.4              ∗              ∗              ∗            = 7.14 molar
                       ml Solución 100 g Solución 98 g H 2SO 4 1 Solución

b) La molalidad de la solución de C12H22O11, se puede determinar a partir de la composición
centesimal, vale decir:

                          19 g C12 H 22 O11       1000 g H 2O 1 mol C12 H 22 O11
                   m=                         ∗               ∗                    = 0.6859 molal
                             81 g H 2 O            1 kg H 2 O   342 g C12 H 22 O11


La fracción molar se determina calculando previamente los moles del soluto y disolvente:

                                                  1 mol C12 H 22 O11
           n C12 H 22 O11 = 19 g C12 H 22O11 ∗                       = n C12 H 22 O11 = 0.0555 mol C12 H 22 O11
                                                  342 g C12 H 22 O11
                          1 mol H 2O
           81 g H 2 O ∗              = 4.50 mol H 2 O
                          18 g H 2 O

La fracción molar será entonces :
                                                              0.0555
                                              X C12 H 22 O11 =        = 0.012
                                                              4.5555
                                              X H2O    = 1 − 0.012 = 0.988



  COMPENDIOS DE QUÍMICA GENERAL PARA ESTUDIANTES DE CIENCIAS E INGENIERÍA
84                          SOLUCIONES Y PROPIEDADES COLIGATIVAS
La molaridad se calcula considerando la densidad de la solución

                          g Solución   19gC12 H 22 O11 1 mol C12 H 22 O11
              M =1.08                 ∗                ∗                    ∗
                          ml Solución   100 g Solución   342 g C12 H 22 O11
              1000 ml Solución
                               = 0.6 molar
                1 Solución


3.3 Una solución acuosa de cloruro de sodio que contiene 8.00 g de NaCl por 100 g de
solución posee una densidad de 1.054 g/ml a la temperatura de 25ºC. Determine: a) la
concentración molar y b) la concentración molal de la solución.
Solución:
                                                   Nº moles de soluto
a) 8 g NaCl/100 g sol.              Molaridad =
                                                   1 litro de solución
ρ = 1.054 g/ml
moles de soluto :
                                          m     8g
                                     n=     =          = 0.137 mol NaCl
                                          M 58.5 g/mol

-Cálculo del volumen en litros de solución:
                                                   1 ml de solución
                                     V = 100 g ∗                    = 94.88 ml
                                                       1.054 g

                                                   V= 0.0949 litros
Molaridad :
                                              0.137 mol NaCl
                                        M=                   = 1.44 molar
                                                 0.0949 
                                                   Nº de moles de soluto
b)                                  Molalidad =
                                                      kg de solvente

Cálculo de la masa en kilogramos de solvente
               Masa del solvente = masa (solución) - masa (soluto) = 100 g - 8 g = 92 g de solvente
                                        Kg de solvente = 92g = 0.092 Kg
                                               0.137 mol de soluto
                                Molalidad =                        = 1.49 molal
                                              0.092 kg de solvente

3.4 Se prepara 1.5 litros de una solución de H2O y C2H5OH, cuya densidad relativa resulta
0.947 en una proporción volumétrica de 3:1 respectivamente. Determinar: a) La densidad del
alcohol etílico. b) La fracción molar del alcohol etílico. c) La molaridad. d) La molalidad.

Solución: Datos:
Vsolución = 1.5 
ρsolución = 0.947 kg/
%VH2O = 75%
%VC2H5OH = 25%

a) VH2O = 1.5  ∗ 0.75 = 1.125 

     COMPENDIOS DE QUÍMICA GENERAL PARA ESTUDIANTES DE CIENCIAS E INGENIERÍA
SOLUCIONES Y PROPIEDADES COLIGATIVAS                              85
     VC2H5OH = 1.5  ∗ 0.25 = 0.375 
                                                                                          kg
                             m sol                         m sol = ρ sol ∗ Vsol = 0.947      ∗ 1.5
                   ρ sol =                                                                 
                             Vsol
                                                           m sol = 1.4205 kg

En consecuencia la masa del alcohol etílico es:
                                                        mC2H5OH = msol – mH2O

                                           mC2H5OH =1.4305 – 1.125 = 0.2955 kg

                                                             mC2H5OH = 295.5 g
La densidad del alcohol etílico es:
                                                             m 295.5g
                                                       ρ=         =         = 0.788g/ml
                                                             V 375ml
                                                                         n C H OH
b)                                                         χ C 2 H 5 OH = 2 5
                                                                            nT

                                                                        295.5 g
                                                   n   C 2 H 5 OH   =           = 6.424 mol
                                                                            g
                                                                        46
                                                                           mol

                                                                     1125 g
                                                       n H 2O =             = 62.5 mol
                                                                         g
                                                                     18
                                                                        mol

                                                     nT = 6.424+62.5= 68.924 mol

                                                                        6.424mol
                                                   χ C2 H 5OH =                   = 0.093
                                                                        68.924mol


c)
                                            n soluto    6.424 C 2 H 5 OHmol
                                      M=              =                     = 4.28 molar
                                            solución      1.5 solución

d)                                              n soluto            6.424 molC 2 H 5 OH
                                      m=                       =                        = 5.71 molal
                                           kg    dilsolvente           1.125 kg H 2 O


3.5 ( 20 puntos, 20 min) La densidad relativa de una disolución acuosa de cloruro de potasio
que contiene 24.6 g KCl es de 1.131 a 21 ºC, mientras que la densidad relativa del cloruro de
potasio sólido, a la misma temperatura es 1.984. Calcular: a) la molaridad, b) la normalidad,
c) la molalidad, d) la fracción molar, y e) el tanto por ciento en volumen. (K = 39; Cl = 35.5)

Solución: Datos:             ρSolución= 1.131 g/ml;           ρKCl= 1.984 g/ml;      ρH2O= 1.0 g/ml;
                               mKCl = 24.6 g                    mH2O = x
                                                             m        m KCl + m H 2O
                                            ρ solución      = total =                 (1)
                                                             Vtotal   V KCl + V H 2O
                                            mH2O = VH2O                                       (2)
                                                   m      24.6 g
                                            V KCl = KCl =        = 12.40ml                    (3)
                                                   ρ KCl 1.984
Reemplazando en (1)
     COMPENDIOS DE QUÍMICA GENERAL PARA ESTUDIANTES DE CIENCIAS E INGENIERÍA
86                              SOLUCIONES Y PROPIEDADES COLIGATIVAS
                                                             24.6 g + m H 2O
                                                   1.131 =
                                                             12.4 + m H 2O
                                                      mH2O = 80.76 g

                                                     VH2O = 80.76 ml
Volumen total:
                                      Vsolución = 12.4 ml + 80.76 ml = 93.16 ml

a) la molaridad es:
                                       24.6 gKCl 1000ml 1molKCl
                                                ∗      ∗           = 3.54molar
                                        93.16ml    1    74.5 gKCl
b) La normalidad es:

               24.6 gKCl 1000ml 1molKCl 1eq − gKCl
                        ∗      ∗           ∗         = 3.54normal
                93.16ml    1    74.5 gKCl   1molKCl
c) La molalidad:

                                     24.6 gKCl 1000 gH 2O 1molKCl
                                               ∗         ∗           = 4.09molal
                                    80.76 gH 2O 1KgH 2O    74.5 gKCl
d) La fracción molar:

                    1molKCl                                              1molH 2 O
      24.6 gKCl ∗             = 0.33molKCl              80.76 gH 2 O ∗               = 4.49molH 2 O
                    74.5 gKCl                                            18 gH 2 O
                               0.33                                      4.49
                     X KCl =        = 0.068                   X H 2O =        = 0.932
                               4.82                                      4.82
                                                        12.4ml
e)     El % en volumen:                    %V KCl =               ∗ 100% = 13.31%
                                                        93.16ml

3.6.- Determinar: a) la concentración molal, b) la fracción molar y la molaridad
de una solución de ácido sulfúrico del 50% en masa y cuya densidad es 1.4 g/ml.

Solución: a) La molalidad de la solución de H2SO4 se puede determinar a partir de la
composición centesimal, vale decir:

                                  50 g H 2SO 4 1000 g H 2O 1 mol H 2SO 4
                            m=                ∗            ∗              = 10.2 molal
                                   50 g H 2 O   1 kg H 2 O   98 g H 2SO 4

La fracción molar se determina calculando previamente los moles del soluto y disolvente:

                                                     1 mol H 2SO 4
                                    50 g H 2SO 4 ∗                 = 0.510 mol H 2SO 4
                                                      98 g H 2SO 4
                                                   1 mol H 2 O
                                    50 g H 2 O ∗               = 2.78 mol H 2 O
                                                   18 g H 2 O
La fracción molar será entonces :
                                                             0.510
                                               X H 2SO 4 =         = 0.155
                                                             3.290
                                               X H2O    = 1 − 0.155 = 0.845
La molaridad se calcula considerando la densidad de la solución
                 g Solución   50 g H 2SO 4   1 mol H 2SO 4 1000 ml Solución
         M = 1.4            ∗              ∗              ∗                 = 7.14 molar
     COMPENDIOSml Solución 100 g Solución 98 g H 2SO 4
                 DE QUÍMICA GENERAL PARA ESTUDIANTES Solución1  DE CIENCIAS E INGENIERÍA
SOLUCIONES Y PROPIEDADES COLIGATIVAS                                           87

3.7 A 100 ml de una disolución de ácido sulfúrico del 96% en masa y de densidad relativa
1.84, se añadieron 400 ml de agua, obteniéndose una solución de peso específico relativo
igual a 1.22. En la solución resultante determinar:
    a) la concentración en tanto por ciento en masa.
    b) La concentración molar
    c) La concentración molal
    d) La normalidad
    e) La fracción molar

Solución: El lector debe hacer énfasis en esquemas de manera que le permitan ver
objetivamente el problema:
                                                                   H2O
                                        V = 100 ml
                                                                     V = ¿?
                                        96%H2SO4
                                                                     %H2SO4 =¿?
                                        ρ = 1.84
                                                                     ρ = 1.22



A 100 ml de una solución de ácido sulfúrico se añade 400 ml de agua, a esta operación se
denomina dilución, que obviamente modificará la concentración y la densidad de la solución.

Para resolver el problema calcularemos el volumen de agua de la solución original para luego
sumar a los 400 ml de agua que se añadieron, a partir de este dato la masa de agua de modo
que sumados a la masa de ácido sulfúrico puro y considerando la densidad de la solución
resultante se podrá determinar el volumen de la solución resultante.

Determinación de la masa de H2O y H2SO4 de la solución original

                                            1.84 g de sol.
                            100 ml de sol. ∗               = 184 g de solución
                                             1 ml de sol.
                                                   96 g H 2SO 4
                            184 g de solución ∗                    = 176.64 g H 2SO 4   la masa de agua en la
                                                100g de solución
                                                                                        solución    resultante
                            m agua = 184 g − 176.64 g = 7.36 g H 2 O
                                                                                        será:

                                    mH2O = (7.36 + 400) g = 407.36 g H2O

De acuerdo a algunas estrategias recomendadas en el presente texto es fundamental conocer la
concentración de la solución resultante en tanto por ciento:

a) El tanto por ciento del ácido sulfúrico se determina a partir de:
m H2SO4 = 176.64 g
m H2O = 407.36 g

La masa total es: (176.64 + 407.36) g = 584 g

                                                   176.64
                                     %H 2SO 4 =           ∗ 100% = 43.36%
                                                   407.36


  COMPENDIOS DE QUÍMICA GENERAL PARA ESTUDIANTES DE CIENCIAS E INGENIERÍA
Propiedades y clasificación de soluciones
Propiedades y clasificación de soluciones
Propiedades y clasificación de soluciones
Propiedades y clasificación de soluciones
Propiedades y clasificación de soluciones
Propiedades y clasificación de soluciones
Propiedades y clasificación de soluciones
Propiedades y clasificación de soluciones
Propiedades y clasificación de soluciones
Propiedades y clasificación de soluciones
Propiedades y clasificación de soluciones
Propiedades y clasificación de soluciones
Propiedades y clasificación de soluciones
Propiedades y clasificación de soluciones
Propiedades y clasificación de soluciones
Propiedades y clasificación de soluciones
Propiedades y clasificación de soluciones
Propiedades y clasificación de soluciones
Propiedades y clasificación de soluciones
Propiedades y clasificación de soluciones
Propiedades y clasificación de soluciones
Propiedades y clasificación de soluciones

Más contenido relacionado

La actualidad más candente

Obtención de Cloruro de t-butilo
Obtención de Cloruro de t-butiloObtención de Cloruro de t-butilo
Obtención de Cloruro de t-butiloAngy Leira
 
Tablas de polaridad de solventes organicos
Tablas de polaridad de solventes organicosTablas de polaridad de solventes organicos
Tablas de polaridad de solventes organicosConalep Ciudad Azteca
 
Muestreo Para AnáLisis QuíMico.2
Muestreo Para AnáLisis QuíMico.2Muestreo Para AnáLisis QuíMico.2
Muestreo Para AnáLisis QuíMico.2yolichavez
 
Titulación potenciométrica de un ácido poliprótico con una base fuerte
Titulación potenciométrica de un ácido poliprótico con una base fuerteTitulación potenciométrica de un ácido poliprótico con una base fuerte
Titulación potenciométrica de un ácido poliprótico con una base fuerteCarolina Vesga Hernandez
 
Reporte de Práctica-Síntesis y Propiedades del Cloruro de Terc-butilo.
Reporte de Práctica-Síntesis y Propiedades del Cloruro de Terc-butilo.Reporte de Práctica-Síntesis y Propiedades del Cloruro de Terc-butilo.
Reporte de Práctica-Síntesis y Propiedades del Cloruro de Terc-butilo.Irvin de Jesús Rodríguez Martínez
 
PRÁCTICA IDENTIFICACIÓN DE CARBOHIDRATOS
PRÁCTICA IDENTIFICACIÓN DE CARBOHIDRATOSPRÁCTICA IDENTIFICACIÓN DE CARBOHIDRATOS
PRÁCTICA IDENTIFICACIÓN DE CARBOHIDRATOSandrea vazquez celio
 
Organica 1 practica 4 determinacion del punto de fusion
Organica 1 practica 4 determinacion del punto de fusionOrganica 1 practica 4 determinacion del punto de fusion
Organica 1 practica 4 determinacion del punto de fusionPeterr David
 
Separacion de los_componentes
Separacion de los_componentesSeparacion de los_componentes
Separacion de los_componentesJ M
 
Tablas de Constantes de Producto de Solubilidad (Kps)
Tablas de Constantes de Producto de Solubilidad (Kps)Tablas de Constantes de Producto de Solubilidad (Kps)
Tablas de Constantes de Producto de Solubilidad (Kps)adriandsierraf
 
TRANSFERENCIA DE MASA
TRANSFERENCIA DE MASATRANSFERENCIA DE MASA
TRANSFERENCIA DE MASANanny Alvarez
 
Oxidación de n-Butanol a n-Butiraldehido
Oxidación de n-Butanol a n-ButiraldehidoOxidación de n-Butanol a n-Butiraldehido
Oxidación de n-Butanol a n-ButiraldehidoJacqueline Vergara
 

La actualidad más candente (20)

Obtención de Cloruro de t-butilo
Obtención de Cloruro de t-butiloObtención de Cloruro de t-butilo
Obtención de Cloruro de t-butilo
 
Tablas de polaridad de solventes organicos
Tablas de polaridad de solventes organicosTablas de polaridad de solventes organicos
Tablas de polaridad de solventes organicos
 
Reporte de Práctica-Obtención y Propiedades del Acetileno.
Reporte de Práctica-Obtención y Propiedades del Acetileno.Reporte de Práctica-Obtención y Propiedades del Acetileno.
Reporte de Práctica-Obtención y Propiedades del Acetileno.
 
Muestreo Para AnáLisis QuíMico.2
Muestreo Para AnáLisis QuíMico.2Muestreo Para AnáLisis QuíMico.2
Muestreo Para AnáLisis QuíMico.2
 
Titulación potenciométrica de un ácido poliprótico con una base fuerte
Titulación potenciométrica de un ácido poliprótico con una base fuerteTitulación potenciométrica de un ácido poliprótico con una base fuerte
Titulación potenciométrica de un ácido poliprótico con una base fuerte
 
Cristalización
CristalizaciónCristalización
Cristalización
 
Reporte de Práctica-Síntesis y Propiedades del Cloruro de Terc-butilo.
Reporte de Práctica-Síntesis y Propiedades del Cloruro de Terc-butilo.Reporte de Práctica-Síntesis y Propiedades del Cloruro de Terc-butilo.
Reporte de Práctica-Síntesis y Propiedades del Cloruro de Terc-butilo.
 
PRÁCTICA IDENTIFICACIÓN DE CARBOHIDRATOS
PRÁCTICA IDENTIFICACIÓN DE CARBOHIDRATOSPRÁCTICA IDENTIFICACIÓN DE CARBOHIDRATOS
PRÁCTICA IDENTIFICACIÓN DE CARBOHIDRATOS
 
Organica 1 practica 4 determinacion del punto de fusion
Organica 1 practica 4 determinacion del punto de fusionOrganica 1 practica 4 determinacion del punto de fusion
Organica 1 practica 4 determinacion del punto de fusion
 
Separacion de los_componentes
Separacion de los_componentesSeparacion de los_componentes
Separacion de los_componentes
 
Acido base
Acido baseAcido base
Acido base
 
Tablas de Constantes de Producto de Solubilidad (Kps)
Tablas de Constantes de Producto de Solubilidad (Kps)Tablas de Constantes de Producto de Solubilidad (Kps)
Tablas de Constantes de Producto de Solubilidad (Kps)
 
Cap. 8 Reacción de hidrocarburos
Cap. 8 Reacción de hidrocarburosCap. 8 Reacción de hidrocarburos
Cap. 8 Reacción de hidrocarburos
 
Informe de-hierro-5
Informe de-hierro-5Informe de-hierro-5
Informe de-hierro-5
 
5.disoluciones.
5.disoluciones.5.disoluciones.
5.disoluciones.
 
TRANSFERENCIA DE MASA
TRANSFERENCIA DE MASATRANSFERENCIA DE MASA
TRANSFERENCIA DE MASA
 
íNdice de saponificación
íNdice de saponificacióníNdice de saponificación
íNdice de saponificación
 
Laboratorio 07
Laboratorio 07Laboratorio 07
Laboratorio 07
 
PESO EQUIVALENTE
PESO EQUIVALENTEPESO EQUIVALENTE
PESO EQUIVALENTE
 
Oxidación de n-Butanol a n-Butiraldehido
Oxidación de n-Butanol a n-ButiraldehidoOxidación de n-Butanol a n-Butiraldehido
Oxidación de n-Butanol a n-Butiraldehido
 

Destacado

Destacado (19)

T 05 Proteínas 17 18
T 05 Proteínas 17 18T 05 Proteínas 17 18
T 05 Proteínas 17 18
 
Futból ensayo
Futból ensayoFutból ensayo
Futból ensayo
 
Cap13
Cap13Cap13
Cap13
 
Cinetica Quimica
Cinetica QuimicaCinetica Quimica
Cinetica Quimica
 
3calculoscomunes
3calculoscomunes3calculoscomunes
3calculoscomunes
 
Neutralizacion
NeutralizacionNeutralizacion
Neutralizacion
 
Comic El Aislamiento: Tu Ahorro Invisible
Comic El Aislamiento: Tu Ahorro Invisible Comic El Aislamiento: Tu Ahorro Invisible
Comic El Aislamiento: Tu Ahorro Invisible
 
Propiedades coligativas de las disoluciones
Propiedades coligativas de las disolucionesPropiedades coligativas de las disoluciones
Propiedades coligativas de las disoluciones
 
Comic de energias renovables
Comic de energias renovablesComic de energias renovables
Comic de energias renovables
 
aplicaciones de propiedades coligativas
aplicaciones de propiedades coligativasaplicaciones de propiedades coligativas
aplicaciones de propiedades coligativas
 
Tema 1. Propiedades Coligativas de Soluciones
Tema 1. Propiedades Coligativas de SolucionesTema 1. Propiedades Coligativas de Soluciones
Tema 1. Propiedades Coligativas de Soluciones
 
Guia operaciones unitarias 3
Guia operaciones unitarias 3Guia operaciones unitarias 3
Guia operaciones unitarias 3
 
Problemas quimica analitica resueltos
Problemas quimica analitica resueltosProblemas quimica analitica resueltos
Problemas quimica analitica resueltos
 
Solubilidad Y ConcentracióN
Solubilidad Y ConcentracióNSolubilidad Y ConcentracióN
Solubilidad Y ConcentracióN
 
Unidades químicas de concentración
Unidades químicas de concentraciónUnidades químicas de concentración
Unidades químicas de concentración
 
CALCULAR CONCENTRACIÓN DE SOLUCIONES QUÍMICAS Lic. Javier Cucaita
CALCULAR CONCENTRACIÓN DE SOLUCIONES QUÍMICAS Lic. Javier CucaitaCALCULAR CONCENTRACIÓN DE SOLUCIONES QUÍMICAS Lic. Javier Cucaita
CALCULAR CONCENTRACIÓN DE SOLUCIONES QUÍMICAS Lic. Javier Cucaita
 
Propiedades Coligativas De Soluciones Quimicas
Propiedades Coligativas De Soluciones QuimicasPropiedades Coligativas De Soluciones Quimicas
Propiedades Coligativas De Soluciones Quimicas
 
Guia
GuiaGuia
Guia
 
Historieta Del Agua
Historieta Del AguaHistorieta Del Agua
Historieta Del Agua
 

Similar a Propiedades y clasificación de soluciones

Presentación1 soluciones m, n y m
Presentación1 soluciones m, n y mPresentación1 soluciones m, n y m
Presentación1 soluciones m, n y mLuis guaman conde
 
BLOQUE II Química II.pptx
BLOQUE II Química II.pptxBLOQUE II Química II.pptx
BLOQUE II Química II.pptxLIMSHOU
 
Disoluciones qm 2010
Disoluciones qm 2010Disoluciones qm 2010
Disoluciones qm 2010Ciencia SOS
 
Soluciones Feb10
Soluciones Feb10Soluciones Feb10
Soluciones Feb10Oli303
 
Diversidad de la materia
Diversidad de la materiaDiversidad de la materia
Diversidad de la materiaantorreciencias
 
Unidades de concentración iii medio
Unidades de concentración iii medio Unidades de concentración iii medio
Unidades de concentración iii medio cravargasp
 
Soluciones, coloides, solubiidad y suspenciones
Soluciones, coloides, solubiidad y suspencionesSoluciones, coloides, solubiidad y suspenciones
Soluciones, coloides, solubiidad y suspencionesdaaladier
 
Unidad ii disoluciones quimicas
Unidad ii  disoluciones quimicasUnidad ii  disoluciones quimicas
Unidad ii disoluciones quimicaspatricio navarrete
 
Soluciones 130817171613-phpapp01
Soluciones 130817171613-phpapp01Soluciones 130817171613-phpapp01
Soluciones 130817171613-phpapp01Manuel Marcelo
 
10. coloides y suspensiones 2013
10. coloides y suspensiones 201310. coloides y suspensiones 2013
10. coloides y suspensiones 201359927103
 
Ii. sistemas dispersos
Ii. sistemas dispersosIi. sistemas dispersos
Ii. sistemas dispersoshzerimar
 

Similar a Propiedades y clasificación de soluciones (20)

SISTEMAS DISPERSOS
SISTEMAS DISPERSOSSISTEMAS DISPERSOS
SISTEMAS DISPERSOS
 
Presentación1 soluciones m, n y m
Presentación1 soluciones m, n y mPresentación1 soluciones m, n y m
Presentación1 soluciones m, n y m
 
BLOQUE II Química II.pptx
BLOQUE II Química II.pptxBLOQUE II Química II.pptx
BLOQUE II Química II.pptx
 
Soluciones,mezclas y coloides
Soluciones,mezclas y coloidesSoluciones,mezclas y coloides
Soluciones,mezclas y coloides
 
Disoluciones qm 2010
Disoluciones qm 2010Disoluciones qm 2010
Disoluciones qm 2010
 
Soluciones Feb10
Soluciones Feb10Soluciones Feb10
Soluciones Feb10
 
Diversidad de la materia
Diversidad de la materiaDiversidad de la materia
Diversidad de la materia
 
Modulo disoluciones 2 medio
Modulo disoluciones 2 medioModulo disoluciones 2 medio
Modulo disoluciones 2 medio
 
Modulo disoluciones 2 medio
Modulo disoluciones 2 medioModulo disoluciones 2 medio
Modulo disoluciones 2 medio
 
Disoluciones
DisolucionesDisoluciones
Disoluciones
 
Coloides
ColoidesColoides
Coloides
 
Unidades de concentración iii medio
Unidades de concentración iii medio Unidades de concentración iii medio
Unidades de concentración iii medio
 
Soluciones, coloides, solubiidad y suspenciones
Soluciones, coloides, solubiidad y suspencionesSoluciones, coloides, solubiidad y suspenciones
Soluciones, coloides, solubiidad y suspenciones
 
DISOLUCIONES.pdf
DISOLUCIONES.pdfDISOLUCIONES.pdf
DISOLUCIONES.pdf
 
Unidad ii disoluciones quimicas
Unidad ii  disoluciones quimicasUnidad ii  disoluciones quimicas
Unidad ii disoluciones quimicas
 
Soluciones
SolucionesSoluciones
Soluciones
 
Soluciones 130817171613-phpapp01
Soluciones 130817171613-phpapp01Soluciones 130817171613-phpapp01
Soluciones 130817171613-phpapp01
 
10. coloides y suspensiones 2013
10. coloides y suspensiones 201310. coloides y suspensiones 2013
10. coloides y suspensiones 2013
 
Ii. sistemas dispersos
Ii. sistemas dispersosIi. sistemas dispersos
Ii. sistemas dispersos
 
Ii. sistemas dispersos
Ii. sistemas dispersosIi. sistemas dispersos
Ii. sistemas dispersos
 

Más de EDWIN QUISBERT

Más de EDWIN QUISBERT (14)

Trabajo, EnergíA Y Potencia
Trabajo, EnergíA Y PotenciaTrabajo, EnergíA Y Potencia
Trabajo, EnergíA Y Potencia
 
Trabajo, EnergíA Y Potencia
Trabajo, EnergíA Y PotenciaTrabajo, EnergíA Y Potencia
Trabajo, EnergíA Y Potencia
 
Final De La Evaluacion De Tecno
Final De La Evaluacion De TecnoFinal De La Evaluacion De Tecno
Final De La Evaluacion De Tecno
 
Hugo DináMica
Hugo DináMicaHugo DináMica
Hugo DináMica
 
Hugo DináMica
Hugo DináMicaHugo DináMica
Hugo DináMica
 
Hugo DináMica
Hugo DináMicaHugo DináMica
Hugo DináMica
 
Admi
AdmiAdmi
Admi
 
Magnitudes FíSicas
Magnitudes FíSicasMagnitudes FíSicas
Magnitudes FíSicas
 
Bien Venidos Al Mundo De La FíSica
Bien Venidos Al Mundo De La FíSicaBien Venidos Al Mundo De La FíSica
Bien Venidos Al Mundo De La FíSica
 
RíO Rocha
RíO RochaRíO Rocha
RíO Rocha
 
RíO Rocha
RíO RochaRíO Rocha
RíO Rocha
 
COMO ENSEÑAR QUIMICA
COMO ENSEÑAR QUIMICACOMO ENSEÑAR QUIMICA
COMO ENSEÑAR QUIMICA
 
Motor De Poggendorff
Motor De PoggendorffMotor De Poggendorff
Motor De Poggendorff
 
MOVIMIENTO VERTICAL
MOVIMIENTO VERTICALMOVIMIENTO VERTICAL
MOVIMIENTO VERTICAL
 

Último

Caja de herramientas de inteligencia artificial para la academia y la investi...
Caja de herramientas de inteligencia artificial para la academia y la investi...Caja de herramientas de inteligencia artificial para la academia y la investi...
Caja de herramientas de inteligencia artificial para la academia y la investi...Lourdes Feria
 
MAYO 1 PROYECTO día de la madre el amor más grande
MAYO 1 PROYECTO día de la madre el amor más grandeMAYO 1 PROYECTO día de la madre el amor más grande
MAYO 1 PROYECTO día de la madre el amor más grandeMarjorie Burga
 
Ecosistemas Natural, Rural y urbano 2021.pptx
Ecosistemas Natural, Rural y urbano  2021.pptxEcosistemas Natural, Rural y urbano  2021.pptx
Ecosistemas Natural, Rural y urbano 2021.pptxolgakaterin
 
el CTE 6 DOCENTES 2 2023-2024abcdefghijoklmnñopqrstuvwxyz
el CTE 6 DOCENTES 2 2023-2024abcdefghijoklmnñopqrstuvwxyzel CTE 6 DOCENTES 2 2023-2024abcdefghijoklmnñopqrstuvwxyz
el CTE 6 DOCENTES 2 2023-2024abcdefghijoklmnñopqrstuvwxyzprofefilete
 
CALENDARIZACION DE MAYO / RESPONSABILIDAD
CALENDARIZACION DE MAYO / RESPONSABILIDADCALENDARIZACION DE MAYO / RESPONSABILIDAD
CALENDARIZACION DE MAYO / RESPONSABILIDADauxsoporte
 
Qué es la Inteligencia artificial generativa
Qué es la Inteligencia artificial generativaQué es la Inteligencia artificial generativa
Qué es la Inteligencia artificial generativaDecaunlz
 
Informatica Generalidades - Conceptos Básicos
Informatica Generalidades - Conceptos BásicosInformatica Generalidades - Conceptos Básicos
Informatica Generalidades - Conceptos BásicosCesarFernandez937857
 
proyecto de mayo inicial 5 añitos aprender es bueno para tu niño
proyecto de mayo inicial 5 añitos aprender es bueno para tu niñoproyecto de mayo inicial 5 añitos aprender es bueno para tu niño
proyecto de mayo inicial 5 añitos aprender es bueno para tu niñotapirjackluis
 
Planificacion Anual 2do Grado Educacion Primaria 2024 Ccesa007.pdf
Planificacion Anual 2do Grado Educacion Primaria   2024   Ccesa007.pdfPlanificacion Anual 2do Grado Educacion Primaria   2024   Ccesa007.pdf
Planificacion Anual 2do Grado Educacion Primaria 2024 Ccesa007.pdfDemetrio Ccesa Rayme
 
Sesión de aprendizaje Planifica Textos argumentativo.docx
Sesión de aprendizaje Planifica Textos argumentativo.docxSesión de aprendizaje Planifica Textos argumentativo.docx
Sesión de aprendizaje Planifica Textos argumentativo.docxMaritzaRetamozoVera
 
TECNOLOGÍA FARMACEUTICA OPERACIONES UNITARIAS.pptx
TECNOLOGÍA FARMACEUTICA OPERACIONES UNITARIAS.pptxTECNOLOGÍA FARMACEUTICA OPERACIONES UNITARIAS.pptx
TECNOLOGÍA FARMACEUTICA OPERACIONES UNITARIAS.pptxKarlaMassielMartinez
 
Planificacion Anual 4to Grado Educacion Primaria 2024 Ccesa007.pdf
Planificacion Anual 4to Grado Educacion Primaria   2024   Ccesa007.pdfPlanificacion Anual 4to Grado Educacion Primaria   2024   Ccesa007.pdf
Planificacion Anual 4to Grado Educacion Primaria 2024 Ccesa007.pdfDemetrio Ccesa Rayme
 
Estrategia de prompts, primeras ideas para su construcción
Estrategia de prompts, primeras ideas para su construcciónEstrategia de prompts, primeras ideas para su construcción
Estrategia de prompts, primeras ideas para su construcciónLourdes Feria
 
Historia y técnica del collage en el arte
Historia y técnica del collage en el arteHistoria y técnica del collage en el arte
Historia y técnica del collage en el arteRaquel Martín Contreras
 
RETO MES DE ABRIL .............................docx
RETO MES DE ABRIL .............................docxRETO MES DE ABRIL .............................docx
RETO MES DE ABRIL .............................docxAna Fernandez
 
TEMA 13 ESPAÑA EN DEMOCRACIA:DISTINTOS GOBIERNOS
TEMA 13 ESPAÑA EN DEMOCRACIA:DISTINTOS GOBIERNOSTEMA 13 ESPAÑA EN DEMOCRACIA:DISTINTOS GOBIERNOS
TEMA 13 ESPAÑA EN DEMOCRACIA:DISTINTOS GOBIERNOSjlorentemartos
 

Último (20)

Caja de herramientas de inteligencia artificial para la academia y la investi...
Caja de herramientas de inteligencia artificial para la academia y la investi...Caja de herramientas de inteligencia artificial para la academia y la investi...
Caja de herramientas de inteligencia artificial para la academia y la investi...
 
MAYO 1 PROYECTO día de la madre el amor más grande
MAYO 1 PROYECTO día de la madre el amor más grandeMAYO 1 PROYECTO día de la madre el amor más grande
MAYO 1 PROYECTO día de la madre el amor más grande
 
Ecosistemas Natural, Rural y urbano 2021.pptx
Ecosistemas Natural, Rural y urbano  2021.pptxEcosistemas Natural, Rural y urbano  2021.pptx
Ecosistemas Natural, Rural y urbano 2021.pptx
 
Power Point: "Defendamos la verdad".pptx
Power Point: "Defendamos la verdad".pptxPower Point: "Defendamos la verdad".pptx
Power Point: "Defendamos la verdad".pptx
 
el CTE 6 DOCENTES 2 2023-2024abcdefghijoklmnñopqrstuvwxyz
el CTE 6 DOCENTES 2 2023-2024abcdefghijoklmnñopqrstuvwxyzel CTE 6 DOCENTES 2 2023-2024abcdefghijoklmnñopqrstuvwxyz
el CTE 6 DOCENTES 2 2023-2024abcdefghijoklmnñopqrstuvwxyz
 
Sesión de clase: Defendamos la verdad.pdf
Sesión de clase: Defendamos la verdad.pdfSesión de clase: Defendamos la verdad.pdf
Sesión de clase: Defendamos la verdad.pdf
 
CALENDARIZACION DE MAYO / RESPONSABILIDAD
CALENDARIZACION DE MAYO / RESPONSABILIDADCALENDARIZACION DE MAYO / RESPONSABILIDAD
CALENDARIZACION DE MAYO / RESPONSABILIDAD
 
Qué es la Inteligencia artificial generativa
Qué es la Inteligencia artificial generativaQué es la Inteligencia artificial generativa
Qué es la Inteligencia artificial generativa
 
Fe contra todo pronóstico. La fe es confianza.
Fe contra todo pronóstico. La fe es confianza.Fe contra todo pronóstico. La fe es confianza.
Fe contra todo pronóstico. La fe es confianza.
 
Informatica Generalidades - Conceptos Básicos
Informatica Generalidades - Conceptos BásicosInformatica Generalidades - Conceptos Básicos
Informatica Generalidades - Conceptos Básicos
 
Medición del Movimiento Online 2024.pptx
Medición del Movimiento Online 2024.pptxMedición del Movimiento Online 2024.pptx
Medición del Movimiento Online 2024.pptx
 
proyecto de mayo inicial 5 añitos aprender es bueno para tu niño
proyecto de mayo inicial 5 añitos aprender es bueno para tu niñoproyecto de mayo inicial 5 añitos aprender es bueno para tu niño
proyecto de mayo inicial 5 añitos aprender es bueno para tu niño
 
Planificacion Anual 2do Grado Educacion Primaria 2024 Ccesa007.pdf
Planificacion Anual 2do Grado Educacion Primaria   2024   Ccesa007.pdfPlanificacion Anual 2do Grado Educacion Primaria   2024   Ccesa007.pdf
Planificacion Anual 2do Grado Educacion Primaria 2024 Ccesa007.pdf
 
Sesión de aprendizaje Planifica Textos argumentativo.docx
Sesión de aprendizaje Planifica Textos argumentativo.docxSesión de aprendizaje Planifica Textos argumentativo.docx
Sesión de aprendizaje Planifica Textos argumentativo.docx
 
TECNOLOGÍA FARMACEUTICA OPERACIONES UNITARIAS.pptx
TECNOLOGÍA FARMACEUTICA OPERACIONES UNITARIAS.pptxTECNOLOGÍA FARMACEUTICA OPERACIONES UNITARIAS.pptx
TECNOLOGÍA FARMACEUTICA OPERACIONES UNITARIAS.pptx
 
Planificacion Anual 4to Grado Educacion Primaria 2024 Ccesa007.pdf
Planificacion Anual 4to Grado Educacion Primaria   2024   Ccesa007.pdfPlanificacion Anual 4to Grado Educacion Primaria   2024   Ccesa007.pdf
Planificacion Anual 4to Grado Educacion Primaria 2024 Ccesa007.pdf
 
Estrategia de prompts, primeras ideas para su construcción
Estrategia de prompts, primeras ideas para su construcciónEstrategia de prompts, primeras ideas para su construcción
Estrategia de prompts, primeras ideas para su construcción
 
Historia y técnica del collage en el arte
Historia y técnica del collage en el arteHistoria y técnica del collage en el arte
Historia y técnica del collage en el arte
 
RETO MES DE ABRIL .............................docx
RETO MES DE ABRIL .............................docxRETO MES DE ABRIL .............................docx
RETO MES DE ABRIL .............................docx
 
TEMA 13 ESPAÑA EN DEMOCRACIA:DISTINTOS GOBIERNOS
TEMA 13 ESPAÑA EN DEMOCRACIA:DISTINTOS GOBIERNOSTEMA 13 ESPAÑA EN DEMOCRACIA:DISTINTOS GOBIERNOS
TEMA 13 ESPAÑA EN DEMOCRACIA:DISTINTOS GOBIERNOS
 

Propiedades y clasificación de soluciones

  • 1. SOLUCIONES Y PROPIEDADES COLIGATIVAS EXIJA SU LIBRO 3.1 DEFINICIÓN DE SOLUCIÓN M uy pocos materiales que encontramos en la vida diaria son sustancias puras; la mayor parte son mezclas y muchas de ellas son homogéneas, Recordemos que las mezclas homogéneas se denominan soluciones o disoluciones, por ejemplo: el aire que respiramos es una mezcla homogénea de varias sustancias gaseosas. El latón es una solución de cinc en cobre. Los océanos son una solución de muchas sustancias que se han disuelto en agua. Una solución es una mezcla homogénea de dos o mas sustancias puras, denominadas componentes de la solución, las cuales pueden ser gaseosas, líquidas o sólidas; debido a que las soluciones líquidas son las mas comunes, en esta sección enfocaremos nuestro estudio en dicho estado. Sin embargo, el estado físico de una solución lo determina a menudo el estado de su componente principal, denominado disolvente. El otro u otros componentes se denominan soluto. Soluto es la sustancia que se Disolvente o solvente, es la sustancia disuelve a través del disolvente en donde se disuelve el llamado soluto. forma de moléculas o iones para Generalmente es aquella que se formar una sola fase. En una encuentra en mayor cantidad. En toda solución puede existir mas de un solución existe un solo solvente. soluto. AGUA YUPI COMPENDIOS DE QUÍMICA GENERAL PARA ESTUDIANTES DE CIENCIAS E INGENIERÍA
  • 2. 58 SOLUCIONES Y PROPIEDADES COLIGATIVAS En la figura 3.1 es un resumen de la relación entre los materiales (materia), recuerde que las sustancias pueden ser mezcladas por procesos físicos y se pueden usar otros procesos físicos para separar la mezcla en 1sustancias. 3.2 PSEUDO - SOLUCIÓN, SUSPENSIÓN Y materia COLOIDE C uando una sustancia se disuelve o dispersa a través de otra, formaremos una mezcla donde encontramos tres posibilidades diferentes de sustancias Mezclas mezclas tamaños de partículas. Cada uno de estos casos elementos homogéneas (soluciones) dará lugar a mezclas denominadas: solución, coloide y suspensión, con propiedades diferentes. Mezclas compuestos heterogéneas La tabla 4.1 muestra una clasificación según el tamaño partículas. Figura 3.1 Relación entre los materiales 1. Cuando la sustancia se disuelve en forma de moléculas o iones entonces se denomina solución y los componentes son denominados soluto y disolvente. 2. Cuando la sustancia se dispersa (no es soluble) y permanece firmemente dividida se denomina coloide y sus componentes son llamados fase dispersa y fase dispersante. 3. Cuando el tamaño de la sustancia a quién se le denomina realmente micela, es mas grande en comparación a los dos casos anteriores se denominará suspensión. TABLA 3.1 Clasificación de la mezclas según el tamaño de miscela SOLUCION COLOIDE SUSPENSIÓN TAMAÑO DE MISCELAS 1-10Å 10-10000Å >10000Å 3.2.1 Coloides hidrofílicos e hidrofóbicos C oloide es una pseudo – solución, es una mezcla heterogénea en la cual las partículas del soluto no precipitan, se dispersan a través de una fase dispersante, se clasifican en dos clases principales: coloides hidrofílicos y coloides hidrofóbicos. Un coloide hidrofílico (que aman el agua) es un coloide en el cual hay una atracción fuerte entre la fase dispersa y la fase continua (agua). Muchos de estos coloides consisten en macromoléculas (moléculas muy grandes) dispersas en agua. Excepto por el gran tamaño de las moléculas dispersas, estos coloides parecen soluciones normales. Un coloide hidrofóbico (que rechazan el agua), es un coloide en el cual hay una falta de atracción entre la fase dispersa y la fase continua (agua) Los coloides hidrofóbicos son básicamente inestables. 1 1 Ebbing Darrell D. Según este autor una sustancia (o sustancia pura) es una clase de materia que no puede ser separada en otras clases de materia por ningún proceso físico. COMPENDIOS DE QUÍMICA GENERAL PARA ESTUDIANTES DE CIENCIAS E INGENIERÍA
  • 3. SOLUCIONES Y PROPIEDADES COLIGATIVAS 59 La tabla 3.2 muestra una clasificación de 8 tipos de coloides que son producto de la mezcla entre los estados sólido, líquido y gaseoso, pero respetando el orden para identificar a la fase dispersa y fase dispersante. No se incluye la mezcla gas-gas porque estos se componen de moléculas individuales. TABLA 3.2 Clasificación de los coloides según el estado de agregación FASE FASE TIPO DE EJEMPLO DISPERSA DISPERSANTE COLOIDE Gas Gas ................. .................. Líquido Gas Aerosol líquido Niebla Sólido Gas Aerosol sólido Humo Gas Líquido Espuma Espuma de cerveza Líquido Líquido Emulsión Leche Sólido Líquido Sol y gel Pintura, jabón en agua Gas Sólido Espuma sólida Esponja, piedra pómez Líquido Sólido Emulsión sólida Mantequilla Sólido Sólido Sol sólida Porcelana Una de las formas prácticas que permite identificar a los coloides, es el hecho de aprovecha el efecto Tyndall, donde se refleja la luz mucho mas intensa en una suspensión que en un coloide, en una solución no hay dicho reflejo. 3.3 CLASIFICACIÓN DE LAS Haz de luz no visible SOLUCIONES L as soluciones se pueden clasificar, atendiendo a 5 aspectos importantes: Fuente luminosa Haz de luz no visible A B C 3.3.1 Según el número de componentes  Soluciones binarias: de dos Haz de luz visible Haz de luz visible componentes Figura 3.2  Soluciones ternarias: de tres Dispersión de un haz de luz por partículas componentes coloidales conocido como efecto Tyndall. La  Soluciones cuaternarias: de cuatro presencia de partículas coloidales pueden componentes detectarse con facilidad con ayuda de un haz de luz. A: Oro coloidal, una solución dorada 3.3.2 Según la naturaleza del disolvente B: Solución de NaCl C: Humo de tabaco, Un aerosol  Soluciones acuosas: el disolvente es el agua  Soluciones orgánicas: El disolvente puede ser: benceno, acetona, etc 3.3.3 Según la naturaleza del soluto  Soluciones ácidas: presencia de sustancias ácidas: HCl, HNO3, etc.  Soluciones básicas: presencia de sustancias básicas: NaOH, Ca(OH)2 COMPENDIOS DE QUÍMICA GENERAL PARA ESTUDIANTES DE CIENCIAS E INGENIERÍA
  • 4. 60 SOLUCIONES Y PROPIEDADES COLIGATIVAS  Soluciones neutras: presencia de sales neutras: NaCl, KNO3, etc. 3.3.4 De acuerdo a la cantidad de sus componentes  Soluciones diluidas: Cuando la masa del soluto es pequeña con respecto al volumen total de la solución.  Soluciones concentradas: Es aquella donde la cantidad del soluto es grande respecto al volumen total de la solución.  Soluciones saturadas: Es aquella solución que ha alcanzado su máxima concentración a una temperatura determinada, es decir que no admite mas soluto porque este comienza a precipitar.  Soluciones sobresaturadas: Es cuando se disuelve mas soluto que la solución saturada debido a ciertos factores mecánicos, por ejemplo la agitación donde se encuentra en equilibrio inestable. 3.3.5 Según los estados de agregación de la materia  Soluciones sólidas: Donde sus componentes se hallan en el estado sólido.  Soluciones liquidas: Donde sus componentes están estado líquido.  Soluciones gaseosas: Donde sus componentes están en estado gaseoso. Ejemplo 3.1.- Dar un ejemplo de una solución constituida por las siguientes fases a) gas-gas, b) líquido-gas, c) líquido-líquido, d) líquido-sólido, e) sólido-sólido. Solubilidad en g soluto/100 g agua Solución: a) Gas- gas : Aire (O2, CO2, etc en N2) b) Líquido – gas: Soda ( CO2 (g) en H2O ()) c) Líquido – líquido: alcohol para fricciones (H2O () en i-C3H7OH ()) d) Líquido – sólido: agua de mar (NaCl, LiCl, etc en H2O ()) KI 200 e) Sólido-sólido: “oro” de 14 180 quilates (Cu en Au) 160 KNO3 3.4 SOLUBILIDAD 140 L 120 a solubilidad se define como la cantidad de una KBr 100 sustancia que se disuelve en una cantidad dada de disolvente (por ejemplo agua) a una temperatura 80 KCl dada para dar una solución saturada; es la 60 concentración máxima posible. Por ejemplo, la sal 40 NaCl de cocina tiene una solubilidad de 36 g por 100 g de 20 K2SO4 agua; por lo tanto, al añadir 40 gramos de NaCl, quedaran 3 g de sal sin disolverse. 10 20 30 40 50 60 70 80 90 100 Temperatura en grados Celcius COMPENDIOS DE QUÍMICA GENERAL PARA ESTUDIANTES 3.3 CIENCIAS E INGENIERÍA Figura DE Efecto de la temperatura en las solubilidades de algunas sales.
  • 5. SOLUCIONES Y PROPIEDADES COLIGATIVAS 61 3.4.1 Factores que afectan la solubilidad L a solubilidad de un sólido es muy sensible a los cambios de temperatura; pero los cambios ordinarios de presión no producen ninguna variación significativa. La figura 3.3 muestra cómo la temperatura afecta la solubilidad de algunos sólidos corrientes en agua, por ejemplo la sacarosa, el nitrato de amonio y el hidróxido de potasio ilustran muy bien este fenómeno; por el contrario, la solubilidad del NaCl casi no se ve afectada por un cambio de temperatura. En algunos casos es posible lograr concentraciones superiores a las concentraciones de saturación, y entonces se tiene una solución sobresaturada. Una solución sobresaturada se puede producir enfriando una solución concentrada caliente. Los compuestos iónicos son solubles en agua, alcohol metílico y amoniaco líquido; e insolubles en octano, benceno y tetracloruro de carbono. Las moléculas de agua, alcohol metílico y amoniaco son polares; cada molécula δ- δ- posee centros positivos y negativos de carga O H O δ+ δ + δ+ eléctrica como muestra la figura 3.4. Otros H H C H líquidos polares también actúan eficientemente AGUA en la disolución de compuestos iónicos. Las H H moléculas de octano y benceno son no polares N ALCOHOL METILICO (ver figura 3.5). Puesto que las electronegatividades del carbono y del hidrógeno H H es muy pequeña, cualquier enlace entre estos dos H AMONIACO átomos es no polar. El enlace entre el cloro y el carbono sí es polar, como se puede deducir al Figura 3.4 estudiar las 2electronegatividades de la tabla 1.3 Algunas moléculas polares del capítulo 1 (página 20) Por regla general los líquidos no polares son ineficientes en la disolución de compuestos iónicos. También encontramos que un líquido polar disolverá otros compuestos polares. El amoniaco y el alcohol metílico se disuelven en agua. La sacarosa, sólido polar (no iónico), es soluble en agua y alcohol metílico, e insoluble en benceno, octano y tetracloruro de carbono. Los disolventes no polares disuelven compuestos no polares. Las grasas y aceites, hidrocarburos no polares, se disuelven en cualquiera de los tres solventes no polares que hemos discutido previamente, pero son virtualmente insolubles en los tres disolventes polares. La regla que se sigue en la elección de H Cl disolventes dice: “ sustancias similares C disuelven sustancias similares”. Los H C C H C disolventes polares disuelven compuestos iónicos y polares; y los no polares disuelven H C C H Cl Cl compuestos no polares. C Cl H 3.4.2 Solubilidad de los gases TETRACLORURO BENCENO DE CARBONO L os gases son muy poco solubles en agua y otros líquidos corrientes. La solubilidad de un gas puede acrecentar aumentando la presión Figura 3.5 Algunas moléculas no polares 2 La electronegatividad de un elemento mide la tendencia relativa del átomo a atraer los electrones hacia sí cuando se combina químicamente con otro átomo ( véase capítulo 1 ). COMPENDIOS DE QUÍMICA GENERAL PARA ESTUDIANTES DE CIENCIAS E INGENIERÍA
  • 6. 62 SOLUCIONES Y PROPIEDADES COLIGATIVAS sobre el gas que se halla por encima de la solución. La solubilidad del oxígeno a diferentes presiones aparece en la tabla 4.3. Es obvio que al observar la última columna, que la solubilidad es directamente proporcional a la presión del gas. Este comportamiento es normal para soluciones de gases en líquidos. TABLA 3.3 Solubilidad del oxígeno en agua a 25 ºC P [mmHg] Solubilidad, [moles O2 m La solubilidad de los gases en los por litro de H2O] ∗10 6 = constante líquidos, al contrario de la de los P 175 0.000307 1.75 sólidos o líquidos, disminuye a medida 300 0.000500 1.67 que aumenta la temperatura. Un 414 0.000688 1.66 químico pondrá a hervir una muestra 610 0.00100 1.64 de agua para reducir la concentración 760 0.00128 1.68 de gases atmosféricos disueltos. Puesto que la solubilidad del oxígeno en agua se reduce considerablemente con el calor, al vaciar agua caliente en los ríos y lagos puede causar serio perjuicio a la vida acuática. TABLA 3.4: Efecto de la temperatura sobre la solubilidad• de los gases en agua•• Gas 0 ºC 10 ºC 50 ºC 100 ºC • La solubilidad se expresa en moles del gas disueltas por H2 0.000960 0.000873 0.000717 0.000715 litro de agua N2 0.00105 0.000830 0.000485 0.000423 •• La presión del gas encima de la solución es 1 atm. O2 0.00212 0.00170 0.000933 0.000758 El efecto de la temperatura sobre la solubilidad CO2 0.0765 0.0533 0.0194 de los gases se muestra en la información de la tabla 3.4. La solubilidad del CO2 es mucho mayor que la de los otros tres gases, debido a que el CO2 reacciona con el agua para forma ácido carbónico: CO2 + H2O ⇒ H2CO3 3.5 EL PROCESO DE DISOLUCIÓN U na sustancia puede disolverse con o sin reacción química en el disolvente. El sodio metálico se disuelve en agua desprendiendo burbujas de hidrógeno y cantidades considerables de calor, se produce un cambio químico en el cual el H2 y el hidróxido de sodio iónico soluble, NaOH, son los productos, la ecuación iónica total será: 2Na(s) + 2H2O → 2[Na+(ac) + OH-(ac)] + H2(g) (1) El cloruro de sodio sólido, por otra parte, se disuelve en agua sin evidencia de reacción química: NaCl(s) → Na+(ac) + Cl-(ac) (2) Si la primera solución (1) se evapora a sequedad, se obtiene hidróxido de sodio sólido, NaOH, en lugar de sodio sólido metálico. Esto junto con los productos de burbujas de hidrógeno, indica que se efectúa una reacción con el disolvente. La evaporación de la solución de cloruro de sodio (2) permite obtener en NaCl original. La facilidad del proceso de disolución depende de dos factores: a) el cambio de energía (reacción exotérmica o endotérmica) y b) el cambio COMPENDIOS DE QUÍMICA GENERAL PARA ESTUDIANTES DE CIENCIAS E INGENIERÍA
  • 7. SOLUCIONES Y PROPIEDADES COLIGATIVAS 63 3 de desorden (entropía) que acompaña al proceso, es decir el proceso de disolución procede hacia la disminución de la energía del sistema, lo cual corresponde a un proceso exotérmico y hacia un incremento de desorden del sistema. El primer factor que se refiere al cambio de energía se denomina calor de solución, ∆Hsolución. En un líquido puro, las fuerzas intermoleculares se producen todas entre moléculas similares; al mezclar un líquido con un sólido, cada molécula experimenta fuerza procedente de otras moléculas o iones diferentes y también de moléculas similares. Las fuerzas relativas de estas interacciones ayudan a determinar el grado de solubilidad de un soluto en un disolvente. Las principales interacciones que afectan la disolución de un soluto en disolvente son: 1. Atracciones soluto-soluto 2. Atracciones disolvente-disolvente 3. Atracciones disolvente-soluto La disolución se favorece cuando el valor de los dos primeros incisos son relativamente pequeños y el del tercero es relativamente grande. Es preciso vencer las atracciones intermoleculares o interiónicas entre las partículas de soluto para que se disuelva, esta parte del proceso requiere consumo de energía. La separación de moléculas de disolvente también consume energía. Sin embargo, cuando las partículas de soluto y las moléculas del disolvente interaccionan en la solución se libera energía y el proceso de disolución es exotérmico. Muchos sólidos se disuelven en líquidos por procesos 4endotérmicos. La razón de que estos procesos se produzcan es que la endotermicidad es contrarrestada por un mayor incremento en el desorden del soluto que acompaña al proceso de disolución. Casi todos los procesos de disolución están acompañados de un incremento de desorden tanto en el disolvente como en el soluto. Por tanto, este factor de desorden suele ser favorable a la solubilidad. 3.6 UNIDADES DE CONCENTRACIÓN L a concentración de un soluto es la cantidad de soluto disuelto en una cantidad dada de disolvente o de solución. La cantidad de disolvente o de solución se puede expresar en términos de volumen o en términos de masa o de cantidad molar. Así, hay varias formas para expresar la concentración de una solución. La concentración de una solución suele expresarse como masa de soluto por unidad de volumen; pero es mas significativa expresarla en términos de número de moles por unidad de volumen. Los términos de concentración mas importantes utilizados por los químicos son: molaridad, molalidad, normalidad y fracción molar. 3.6.1 Molaridad 3 Whitten Kennet W. define entropía como la medición del desorden de un sistema, mientras mayor sea el desorden de un sistema, mayor será su entropía. 4 Proceso endotérmico es cuando en el proceso de disolución de un soluto, en el disolvente, éste absorbe energía del medio ambiente. COMPENDIOS DE QUÍMICA GENERAL PARA ESTUDIANTES DE CIENCIAS E INGENIERÍA
  • 8. 64 SOLUCIONES Y PROPIEDADES COLIGATIVAS L a concentración molar , o molaridad [M], se define como los moles de soluto disueltos en un litro de solución. moles de soluto Molaridad [ M ] = (3.1) litro de solución Una solución acuosa de HCl que es 0.2 M, contiene 0.2 moles de HCl por litro de solución, en la práctica si se quiere preparar esta solución, se agrega a un matraz volumétrico aforado de 1.00  una cierta cantidad de agua por ejemplo ¼ , al cual añadimos 0.2 moles de HCl ( debe hacerse cálculos a partir de los datos que lleva la etiqueta del frasco). En seguida se llena el matraz con agua adicional a la marca hasta el cuello y se mezcla la solución. Ejemplo 3.2.- 0.25 g de una muestra de sulfato de cobre pentahidratado (CuSO 4∗5H2O) se coloca en un matraz aforado de 250 cc. El matraz se llena con agua hasta la marca en el cuello. ¿Cuál es la molaridad de la solución resultante? Solución.- Para determinar la molaridad, se necesitan los moles de soluto. Por consiguiente convertiremos los gramos de CuSO4.5H2O a moles: 1 mol CuSO 4 ∗ 5H 2 O n = 0.25 g CuSO 4 ∗ 5H 2 O ∗ = 0.1 mol CuSO 4 ∗ 5H 2 O 249.54 g El volumen de la solución es 0.25 litros, en consecuencia la molaridad es: 0.1 mol CuSO 4 ∗ 5H 2 O Molaridad = = 0.4 M 0.25 litros 3.6.2 Equivalente - gramo de ácidos y bases S e define un equivalente – gramo de un ácido como el peso en gramos de éste que puede producir un mol de H+ y un equivalente – gramo de una base como el peso en gramos de ésta que pueda producir un mol de OH-. El peso equivalente o equivalente - gramo de un ácido se obtiene dividiendo su peso fórmula o peso molecular entre el número de hidrógenos ácidos que aporta una molécula del ácido. El peso equivalente o equivalente - gramo de una base se obtiene dividiendo su peso fórmula o peso molecular entre el número de iones hidróxido. A partir de definiciones anteriores, se ve que un equivalente gramo de cualquier ácido reacciona con un equivalente – gramo de cualquier base. No es cierto que un mol de cualquier ácido reacciona con un mol de cualquier base en una reacción química específica. En consecuencia, según la definición de equivalentes, 1 eq-g ácido = 1 eq-g base. Por lo general, se puede escribir la siguiente expresión para todas las reacciones ácido base que llegan hasta el 5punto final. Eq de ácido = eq de base o meq de ácido = meq de base (3.2) Donde: meq = miliequivalentes, 1 eq = 1000 meq 3.6.3 Equivalente – gramo en reacciones Redox 5 Punto final es el punto en el cual el indicador cambia de color y se define la titulación en una reacción de neutralización. COMPENDIOS DE QUÍMICA GENERAL PARA ESTUDIANTES DE CIENCIAS E INGENIERÍA
  • 9. SOLUCIONES Y PROPIEDADES COLIGATIVAS 65 E n reacciones de oxidación – reducción se debe recordar que un agente oxidante acepta electrones, y un agente reductor los produce. El principio de equivalente permite hacer cálculos estequiométricos en reacciones redox. El equivalente gramo del agente oxidante y del agente reductor es respectivamente: peso molecular Eq − g (agente oxidante) = No. e − ganados peso molecular Eq − g (agente reductor) = No. e − perdidos 3.6.4 Normalidad L a concentración normal , o normalidad [N], de una solución se define como el número de pesos equivalentes o simplemente equivalentes (eq) de soluto por litro de solución. Número de peso equivalente de soluto Normalidad [ N ] = o litro de solución No. de miliequivalente de soluto Normalidad[ N ] = litro de solución (3.3) Ejemplo 3.3 ¿Cuál es la normalidad de una solución que contiene 10 g de H 2SO4 en 500 ml de solución? Solución.- El número de moles de H2SO4 es: 1 mol H 2SO 4 n = 10 g H 2SO 4 ∗ = 0.102 mol H 2SO 4 98 g H 2SO 4 El volumen de la solución es 0.5 litros, en consecuencia la normalidad es: 0.102 moles H 2 SO 4 2eq − gH 2 SO 4 Normalidad = ∗ = 0.408 N 0.5 litros 1molH 2 SO 4 3.6.5 Molalidad L a concentración molal , o molalidad [m], se define como los moles de soluto disueltos en un kilogramo de disolvente. moles de soluto Molalidad [ m] = (3.4) kg de disolvente COMPENDIOS DE QUÍMICA GENERAL PARA ESTUDIANTES DE CIENCIAS E INGENIERÍA
  • 10. 66 SOLUCIONES Y PROPIEDADES COLIGATIVAS Ejemplo 3.4.- El tolueno, C6H5CH3, es un compuesto líquido similar al benceno, C6H6. Es la materia prima para otras sustancias, entre ellas el trinitrotolueno (TNT). Encuentre la molalidad del tolueno en una solución que contiene 5 g de tolueno en 25 g de benceno. Solución.- El número de moles de tolueno es: 1 mol de C 6 H 5 CH 3 5 g C 6 H 5 CH 3 ∗ = 0.0543 mol C 6 H 5 CH 3 92 g C 6 H 5 CH 3 La masa del benceno que constituye el disolvente es: 0.025 kg C6H6 0.0543 mol C 6 H 5 CH 3 mol soluto molalidad = = 2.172 = 2.172 molal 0.025 kg C 6 H 6 kg solvente 3.6.6 Formalidad L a formalidad de una solución es el número de peso fórmula en gramos de soluto por litro de solución; el símbolo de esta unidad es F. La formalidad es muy similar a la molaridad, por ejemplo 1 formal que se simboliza 1F del ácido clorhídrico es igual a su peso fórmula, es decir 36.5 g HCl, por litro de disolución. 3.6.6 Fracción molar L a fracción molar de una sustancia A (χA), componente de una solución se define como los moles de sustancia A divididos entre los moles totales de solución, esto es: moles de sustancia A n χA = = A (3.5) moles totales de la solución n t 3.7 DILUCIÓN E n la práctica de laboratorio con frecuencia se usan soluciones concentradas de concentración conocida para preparar a partir de ellas soluciones diluidas. Por tanto, es importante ver cómo los cambios de volumen afectan la concentración de una solución. Cuando la concentración se expresa en una escala volumétrica, la cantidad de soluto contenido en un volumen determinado de la solución es igual al producto del volumen por la concentración, es decir: Cantidad de soluto disuelto = volumen ∗ concentración Cuando se diluye una solución, el volumen aumenta y la concentración disminuye, pero la cantidad total de soluto permanece constante. Por esta razón, dos soluciones de concentraciones diferentes pero que contengan las mismas cantidades de soluto, están relacionadas por la siguiente expresión: Volumen1 ∗ Concentración1 = Volumen2 ∗ Concentración2 V1 ∗ C1 = V2 ∗ C2 (3.6) Para reacciones de neutralización ácido – base la ecuación correspondiente y de mucha utilidad es: COMPENDIOS DE QUÍMICA GENERAL PARA ESTUDIANTES DE CIENCIAS E INGENIERÍA
  • 11. SOLUCIONES Y PROPIEDADES COLIGATIVAS 67 N1∗V1 = N2∗V2 (3.7) Donde N1 y N2 son concentraciones normales y V1 y V2 son los volúmenes requeridos para la neutralización de una base y una ácido. No obstante se usa también para diluciones. Ejemplo 3.4.- 0.25  de una solución contiene 75 g de NaCl por litro. a) ¿A qué volumen se debe diluir para obtener una solución de concentración 15 g de NaCl por litro?, b) ¿qué cantidad de agua se necesita para este propósito? Solución: a) Considerando la expresión (3.6) y despejando Volumen se tiene: V1 ∗ C1 = V2 ∗ C2 V ∗C 0.25  ∗ 75 g/ V2 = 1 1 = = 1.25  C2 15 g/ b) La cantidad de agua que se requiere para esta dilución es: (1.25 – 0.25)  = 1  Ejemplo 3.5.- Calcule el volumen aproximado del agua que se debe agregar a 250 cm 3 de una solución 1.25 N para preparar una solución de concentración 0.5 N (desprecie los cambios en volumen) Solución: Considerando la expresión (3.7) se puede determinar inicialmente el volumen de la nueva concentración y luego el volumen de agua requerido en la dilución de la concentración original. N1∗V1 = N2∗V2 N1V1 1.25 N ∗ 250 cm 3 V2 = = = 625 cm 3 N2 0.5 N La cantidad de agua es: (625 – 250) cm3 = 375 cm3 Ejemplo 3.6.- Si una solución se prepara a partir de 1 mol de etilenglicol y 9 mol de agua, los moles totales de la solución son 10 moles, en consecuencia la fracción molar del etilenglicol es: 1 mol etilenglicol χ= = 0.1 10 moles Recordemos que la suma de las fracciones de todos los componentes de una solución es igual a 1. n (3.8) X =1∑ i =1 i Algunas estrategias: 3.8 REACCIONES QUÍMICAS EN SOLUCIONES La molaridad y la normalidad se calculan partiendo de la densidad y del porcentaje C omo es de conocimiento, se discutió el método de composición de launa solución. En soluciones diluidas densidad de la para resolver problemas en que intervienen solución es muy semejante a la densidad reacciones químicas, ahora estamos preparados para del disolvente puro. Para calcular la molalidad y fracción molar solo es COMPENDIOS DE QUÍMICA GENERAL PARA ESTUDIANTES DE conocer el E INGENIERÍA necesario CIENCIAS porcentaje de composición de la solución.
  • 12. 68 SOLUCIONES Y PROPIEDADES COLIGATIVAS extender el método a reacciones en solución y poder efectuar cálculos estequiométricos a partir de un volumen de una concentración determinada a la cantidad de soluto presente. Los problemas en esta reacción se basan en dos principios:  El número de equivalentes de todas las especies en una reacción química es el mismo.  Volumen ∗ normalidad = número de equivalentes Ejemplo 3.7 ¿Cuántos mililitros de NaOH 4.0 N se necesitan para neutralizar 20 ml de HCl 3.0 N? Solución: Se puede usar el concepto de miliequivalentes, esto es: 4.0 meq 3.0 meq 4.0 N = 3 y 3.0 N = cm cm 3 Si: N1 = 4.0 N N2 = 3.0 N V2 = 20 ml HCl V1 = ¿? N 2 V2 3.0 ∗ 20 ml V1 = = = 15 ml NaOH N1 4.0 Ejemplo 3.8.- ¿Cuántos gramos de NaOH se necesitan para neutralizar 90 ml de HCl 1.5 M? Solución: Para resolver el problema es conveniente transformar la concentración molar en normalidad, esto es, 1 eq-g HCl = 1 mol HCl En consecuencia: moles HCl 1 eq − g HCl 1.5 ∗ = 1.5 N  1 mol HCl Por tanto, en una reacción ácido base se tiene: No eq HCl = No eq NaOH 1.5 meq HCl 90 ml solución ∗ = 135 meq HCl = 135 meq NaOH 1 ml solución Por consiguiente la masa de NaOH requerida es: 1 eq − g NaOH 40 g NaOH 135 meq NaOH ∗ ∗ = 5.4 g NaOH 1000 meq NaOH 1 eq − g NaOH Ejemplo 3.9.-Encuentre el peso equivalente de KMnO4 en la siguiente reacción: MnSO4 + KMnO4 + H2O ⇔ MnO2 + H2SO4 + K2SO4 b) ¿Cuántos gramos de MnSO4 se oxidan con 50 cm3 de una solución de KMnO4 0.1 N? Solución: Para resolver el problema efectuaremos la igualación de la ecuación química. [Mn+2 + SO4= ] + [K+ + MnO4- ] + H2O0 ⇒ MnO20 + [2H+ + SO4= ] + [2K+ + SO4=] 2∗ 3 e- + 4 H+ + MnO4- ⇒ MnO20 + 2 H2O0 COMPENDIOS DE QUÍMICA GENERAL PARA ESTUDIANTES DE CIENCIAS E INGENIERÍA
  • 13. SOLUCIONES Y PROPIEDADES COLIGATIVAS 69 3∗ 2 H2O0 + Mn+2 ⇒ MnO20 + 4 H+ + 2 e- 2 MnO + 2 H2O + Mn+2 4 - ⇒ 5 MnO20 + 4 H+ La ecuación igualada es: 3 MnSO4 + 2 KMnO4 + 2 H2O ⇔ 5 MnO2 + 2 H2SO4 + K2SO4 a) el peso equivalente del permanganato de potasio es: 158 g eq − g KMnO 4 = = 52.67 g 3 b) La masa de sulfato manganoso requerido es: 0.1 eq KMnO 4 1 mol KMnO 4 3 mol MnSO 4 151 g MnSO 4 50 cm 3 ∗ ∗ ∗ ∗ = 0.3775 g MnSO 4 1000 cm 3 3 eq KMnO 4 2 mol KMnO 4 1 mol MnSO 4 3.9 PROPIEDADES COLIGATIVAS A lgunas propiedades físicas de las soluciones difieren mucho de las del disolvente puro. Por ejemplo, el agua pura se congela a 0 ºC, pero las soluciones acuosas se congelan a temperaturas menores. El etilenglicol se adiciona al agua de los radiadores de los automóviles, pues es un anticongelante ya que abate (disminuye) el punto de congelación de la solución; también eleva el punto de ebullición de la solución sobre la del agua pura, permitiendo que el motor funcione a una temperatura mas alta. Una solución, que conste de dos o mas componentes, carece de las propiedades físicas constantes de una sustancia pura; estas propiedades dependen de la concentración de las partículas del soluto y no de su naturaleza. Tales propiedades se conocen como propiedades coligativas y éstas son: el descenso de la presión de vapor; la depresión en el punto de congelación; la elevación en el punto de ebullición y la presión osmótica. Las propiedades coligativas se pueden usar en la determinación de los pesos moleculares de las sustancias disueltas y pueden dar además información acerca de las propiedades del soluto si se conocen las propiedades del disolvente. Todas las soluciones obedecen las leyes que se discuten en este capítulo , cuando la concentración es suficientemente baja. Por esta razón se les conoce como leyes de las soluciones diluidas. 3.9.1 Descenso de la presión de vapor y la ley de Rault M uchos experimentos han demostrado que las soluciones que contienen líquidos no volátiles o sólidos como solutos, siempre tienen presiones de vapor mas bajas que los disolventes puros. Fig. A FIGURA 3.6 La figura A muestra la presión de vapor del agua pura, la figura B el equilibrio de dos sistemas que contienen agua Fig. B pura, en la figua C la presión manométrica (diferencia de altura) de los dos líquidos se debe al vapor de agua Fig. C pura. Esta presión es menor sobre la solución azucarada, por que hay menos moléculas de agua por unidad de área de COMPENDIOS DE QUÍMICA GENERAL PARA ESTUDIANTES DE CIENCIAS E INGENIERÍA superficie al evaporarse.
  • 14. 70 SOLUCIONES Y PROPIEDADES COLIGATIVAS Cuando un soluto no volátil se disuelve en un líquido; parte del volumen total de la solución es ocupada por moléculas de soluto, y por lo tanto, hay menos moléculas de disolvente por unidad de área en la superficie. Esto ocasiona el descenso de presión de vapor del disolvente. El descenso de la presión de vapor asociados con solutos no volátiles y no ionizables se resume en la ley de Rault: “La presión de vapor de un disolvente en una solución es igual a la presión de vapor del disolvente puro multiplicada por la fracción molar del mismo en la disolución”. Matemáticamente se expresa: P = Xd.Pº (4.9) Donde: Xd, representa la fracción molar del disolvente en la solución, Pº es la presión de vapor del disolvente puro y P es la presión de vapor del disolvente en la solución. Esta ley permite calcular pesos moleculares. Ejemplo 3.10.- La presión de vapor del agua pura a 25 ºC es 23.76 mmHg y la presión de vapor de una mezcla formada por 5.4 g de soluto no volátil en 90 g de agua es 23.32 mmHg. Determinar el peso molecular de dicho soluto. Solución: De acuerdo a la ley de Rault: P = Xd.Pº P 23.32 Xd = = = 0.9815 Po 23.76 Puesto que la fracción molar se define como: moles de sustancia A n XA = = A moles totales de la solución n t Se puede expresar en términos de sus pesos moleculares: md Md 0.9815 = md m + s Md Ms reemplazando datos: 90 g 0.9815 = 18 90 g 5.4 g + 18 Ms despejando Ms: Ms= 57.3 g/mol Ejemplo 3.11.- Determine la presión de vapor a 25 ºC de una solución acuosa que consta de 10 g de sacarosa, C12H22O11 y 75 g de H2O. Solución: De acuerdo a la ley de Rault: COMPENDIOS DE QUÍMICA GENERAL PARA ESTUDIANTES DE CIENCIAS E INGENIERÍA
  • 15. SOLUCIONES Y PROPIEDADES COLIGATIVAS 71 P = Xd.Pº La fracción molar del disolvente es: md Md Xd = md m + s Md Ms 4.17 χd = = 0.993 Por consiguiente La presión de 4.17 + 0.0292 vapor es: Pv = 0.993 ∗ 23.76 mmHg = 243.59 mmHg 3.9.2 La ley de Rault y las soluciones de solutos volátiles E n soluciones en las cuales tanto el disolvente como el soluto tienen una presión de vapor apreciable, se puede aplicar la ley de Rault a ambos componentes: Para lograr cierta comprensión de tales mezclas, considere una solución ideal que contiene dos componentes, A y B, según la ley de Rault: PA = XA.PAº y PB = XB.PBº La presión total del sistema es entonces, la suma de las presiones parciales de cada componente volátil: Ptotal = PA + PB = XA.PAº + XB.PBº (3.10) Ejemplo 3.12.- ¿Cuál es la presión de vapor que ejerce una mezcla formada por 100 gramos de benceno y 100 g de tolueno a 25 ºC? A dicha temperatura las presiones del vapor de benceno y tolueno puros son, respectivamente 95.1 y 28.4 mmHg. Solución: De acuerdo a la expresión (3.6) Ptotal = PC6H6 + PC7H8 P total = XC6H6* PC6H6º + XC7H8 * PC7H8º Las fracciones molares de las dos sustancias son: 100 g 100 g n benceno = = 1.282 n tolueno = = 1.087 g g 78 92 mol mol n total = 1.282 + 1.087 = 2.369 1.282 1.087 χ C6 H 6 = = 0.541 χ C7 H8 = = 0.459 2.369 2.369 La presión de vapor será: Pt = 0.549* 95.1 mmHg + 0.459 * 28.4 mmHg Pt = 52.21 + 13.04 = 65.25 mmHg COMPENDIOS DE QUÍMICA GENERAL PARA ESTUDIANTES DE CIENCIAS E INGENIERÍA
  • 16. 72 SOLUCIONES Y PROPIEDADES COLIGATIVAS Las soluciones pueden representarse gráficamente. La figura 3.7 muestra la ley de Rault para una solución ideal de un soluto en un líquido volátil. La presión de vapor que ejerce el líquido es PRESION DE VAPOR PRESION DE VAPOR DEL DISOLVENTE DEL DISOLVENTE proporcional a su fracción molar en la solución. Este diagrama se cumple para: P = Xd.Pº III La figura 3.8 muestra la ley de Rault aplicado a soluciones que II tienen dos componentes volátiles. Este diagrama se cumple para: I Ptotal = PA + PB = XA.PAº + XB.PBº En la figura 3.8 la línea (I) es la A Xdisolvente B A Xdisolvente B presión parcial de A y la línea (II) Figura 3.7 Figura 3.8 es la presión parcial de B y la línea (III) es la presión total para diferentes concentraciones de los dos líquidos volátiles. La figura 3.9 muestra una desviación negativa de la ley de Rault, Las fuerzas intermoleculares existentes en la solución son superiores a las fuerzas intermoleculares de los componentes puros aisladamente. La figura 3.10 muestra la presión de vapor superior a la presión PRESION DE VAPOR DEL DISOLVENTE predicha por la ley de Rault (desviación positiva). Las fuerzas intermoleculares existentes en la solución son más débiles que las de los componentes puros A Xdisolvente B A Xdisolvente B Figura 3.9 Figura 3.10 independientemente. 3.9.3 LEY DE HENRY E l efecto de la presión sobre la solubilidad de un gas en un líquido se puede predecir de manera cuantitativa. Para soluciones diluidas de un gas no reactivo en un líquido, puede COMPENDIOS DE QUÍMICA GENERAL PARA ESTUDIANTES DE CIENCIAS E INGENIERÍA
  • 17. SOLUCIONES Y PROPIEDADES COLIGATIVAS 73 aplicarse una expresión muy similar a la ley de Rault, conocida como la ley de Henry, cuya expresión matemática es: Pgas = K∗Xgas (3.11) Donde Pgas es la presión del gas sobre la superficie de la solución y k es una constante para un gas y un disolvente determinados a una determinada dada. Xgas representa la fracción molar del gas disuelto; la relación es válida a bajas concentraciones y a bajas presiones. Ejemplo 3.13.- Si 29 mg de N2 se disuelven en un litro de agua a 0 ºC y 760 mmHg de presión de N2, ¿qué masa de N2 se disolverá en un litro de agua a 0 ºC y 5 atm de presión? Solución: De acuerdo con la ecuación (3.11), en principio se determinará la constante de Henry K, para luego determinar la masa de nitrógeno disuelto en las nuevas condiciones de presión y concentración. La fracción molar del gas es: 1g N2 1 mol N 2 29 mg N 2 ∗ ∗ = 1.036 ∗ 10 −3 mol N 2 1000 g N 2 28 g N 2 1000 ml 1 g 1 mol H 2 O 1 H 2 O ∗ ∗ ∗ = 55.56 mol H 2 O 1 H 2 O 1 ml 18 g H 2 O 1.036 ∗ 10 −3 molN 2 χ N2 = = 1.8646 ∗ 10 −5 55.561 La constante de Henry será: Pgas 1 atm K= = = 5.363 ∗10 4 χgas 1.8646 ∗10 −5 La masa de nitrógeno disuelta en 1 litro de agua a 5 atm será: Pgas 5atm χ N2 = = 4 = 9.323 ∗10 −5 K 5.363 ∗10 1 litro de H2O = 55.56 mol nN 2 nN2 χ N2 = = = 9.323 ∗10 −5 n N 2 + n H 2O n N 2 + 55.56 Resolviendo la ecuación se tiene: 28 g N 2 1000 mg N 2 5.18 ∗ 10 −3 mol N 2 ∗ ∗ = 145.04 mg N 2 1 mol N 2 1 g N2 3.9.4 Aumento del punto de ebullición R ecordemos que el punto de ebullición de un líquido es la temperatura a la cual la presión de vapor se iguala a la presión aplicada en su superficie, por ejemplo la temperatura de ebullición normal del agua sobre el nivel del mar es 100 ºC COMPENDIOS DE QUÍMICA GENERAL PARA ESTUDIANTES DE CIENCIAS E INGENIERÍA
  • 18. 74 SOLUCIONES Y PROPIEDADES COLIGATIVAS Se ha visto que la presión de vapor de un disolvente a una temperatura dada, desciende por la presencia de un soluto no volátil. Las soluciones de este tipo deben calentarse a temperaturas mas altas que el disolvente puro para que su presión de vapor iguale a la presión atmosférica. El incremento en el punto de ebullición, ∆Te ( en relación al punto de ebullición del disolvente puro), es directamente proporcional al número de partículas de soluto por mol de disolvente. Sabemos que la molalidad expresa el número fijo de moles de disolvente. Así Te PRESION DE VAPOR es proporcional a la molalidad, como se muestra en la siguiente expresión matemática: DEL DISOLVENTE ∆Te = Kem (3.12) ∆Te = Tf(solución) – Tf(disolvente) Ejemplo 3.14.- Una disolución de glicocola preparada al disolver 1.62 g de sustancia en 19.62 g de agua, hierve a 100.6 ºC. Hallar el peso molecular de la glicocola. (Ke = 0.52 ºC/molal). Solución: Datos: Te = 100.6 ºC Ke = 0.52 ºC/molal De acuerdo a la relación (2.7) ∆Te = Kem Se puede determinar la molalidad ΔTe (100.6 − 100)°C m= = = 1.154 molal ke 0.52 °C/ molal A partir de este dato se puede evaluar el número de moles de soluto: moles de soluto 1.154 ∗ 19.62 g de agua = 0.0226 moles de soluto 1000 g de agua En consecuencia el peso molecular de soluto será: ms 1.62 g M= = = 71.68 g/mol n s 0.0226 moles Para hallar el peso molecular se puede considerar un segundo método propuesto por el Prof. José Ibarz Aznárez, el cuál expresa: Si una disolución está constituida por a gramos de soluto y A g de disolvente, y el peso molecular del soluto es M, la molalidad de disolución es: a • 1000 m= (3.13) A•M Considerando la expresión (3.8), y despejando M se tiene: COMPENDIOS DE QUÍMICA GENERAL PARA ESTUDIANTES DE CIENCIAS E INGENIERÍA
  • 19. SOLUCIONES Y PROPIEDADES COLIGATIVAS 75 a • 1000 1.62 • 1000 M= = = 71.55 g/mol A•m 19.62 • 1.154 3.9.5 Descenso del punto de congelación E n contraste con el punto de ebullición, el punto de congelación de una solución generalmente es mas bajo que el punto de congelación del solvente puro, como muestra la figura 3.6. La diferencia entre estas dos temperaturas se conoce como depresión en el punto de solidificación y se designa por ∆Tc, y es proporcional a la concentración molal del soluto. Esta proporcionalidad, convertida en igualdad se puede expresar por medio de la siguiente ecuación: Tc = Kcm (3.14) ∆Tc = Tf(disolvente) – Tf(solución) Presión de vapor [mmHg] La figura 3.11 muestra como un soluto no volátil abate la presión de vapor de un disolvente, el punto de ebullición se eleva y el punto de congelación desciende con respecto a los puntos correspondientes en los disolventes puros. La magnitud de elevación del punto de ebullición ∆Te es menor que la magnitud del abatimiento del punto de ∆P congelación ∆Tc. Ejemplo 3.15.- Se funde una mezcla de 0.436 g de acetanilida con 14.06 g de alcanfor de punto de fusión 176.2 ºC. La mezcla se deja solidificar y enfriar, se reduce ∆Tc Temperatura [ºC] ∆Te a polvo, se mezcla íntimamente y se Figura 3.11 calienta. En el momento en que se termina Diagrama de fase del H2O y de una solución acuosa de fundir su temperatura es de 167.0 ºC. Hallar el peso molecular de la acetanilida. (kcalcanfor = 40.0 ºC/molal) Solución: Los datos son los siguientes: Tc = 176.2 ºC kc = 40.0 ºC/molal De acuerdo a la relación (3.7) ∆Tc = kcm Se puede determinar la molalidad: ΔTc (176.2 − 167.0)°C m= = = 0.23 molal kc 40.0 °C/ molal COMPENDIOS DE QUÍMICA GENERAL PARA ESTUDIANTES DE CIENCIAS E INGENIERÍA
  • 20. 76 SOLUCIONES Y PROPIEDADES COLIGATIVAS A partir de este dato se puede evaluar el número de moles de soluto: moles de soluto n = 0.23 ∗ 14.06 g de alcanfor 1000 g de alcanfor n = 0.00323 moles de soluto En consecuencia el peso molecular de soluto será: ms 0.436 g M= = = 134.98 g/mol n s 0.00323 moles Para hallar el peso molecular se puede considerar la expresión (3.8), y despejando M se tiene: a • 1000 1.62 • 1000 M= = = 71.55 g/mol A • m 19.62 • 1.154 TABLA 3.2 Algunas propiedades de disolventes comunes PUNTO DE ke PUNTO kc SOLVENTE EBULLICIO [ºC/m] FUSION [ºC/m] N [ºC] [ºC] H2O 100.00 0.52 0.0 1.86 C6H6 80.10 2.53 5.50 5.12 CCl4 76.80 5.02 -22.3 29.80 C2H5OH 78.40 1.22 -114.6 1.99 ClCHO 61.20 3.63 -63.50 4.68 3.9.6 Aplicaciones del aumento y descenso del punto de ebullición y fusión respectivamente Ejemplo 3.16.- El punto de ebullición de una solución de 0.402 g de naftaleno, en 26.6 g de cloroformo, es 0.455 ºC mas alto que el del cloroformo puro. ¿Cuál es la constante ebulloscópica del cloroformo? Solución: A partir de la expresión (2.6) ∆Te = Kem, se puede despejar Ke, puesto que ∆Te = 0.455° y la molalidad es: 0.402 g C10 H 8 1000 g CHCl3 1 mol C10 H 8 m= ∗ ∗ m = 0.118 molal 26.6 g CHCl3 1 kg CHCl 3 128 g C10 H 8 ΔTe 0.455 °C Ke = = = 3.86 °C/molal m 0.118 m Ejemplo 3.17.- La presión de vapor de una solución acuosa diluida es 23.45 torr a 25 ºC, mientras que la presión de vapor del agua pura a la misma temperatura es 23.76 torr. Calcúlese la concentración molal del soluto, y utilícense los valores tabulados de Ke del agua para predecir el punto de ebullición de la solución. Solución: Puesto que la concentración molal se define como: COMPENDIOS DE QUÍMICA GENERAL PARA ESTUDIANTES DE CIENCIAS E INGENIERÍA
  • 21. SOLUCIONES Y PROPIEDADES COLIGATIVAS 77 moles de soluto Molalidad [ m] = kg de disolvente se puede considerar la masa del disolvente como 1 kg = 1000 g H2O. De acuerdo a la ley de Rault: P = Xd.Pº P 23.45 Xd = o = = 0.987 P 23.76 Puesto que la fracción molar se define como: moles de sustancia A n XA = = A moles totales de la solución n t Se puede expresar en términos de sus pesos moleculares: md Md 0.987 = md m + s Md Ms reemplazando datos: 1000 g 0.987 = 18 1000 g + ns 18 El número de moles del soluto, despejando de la anterior expresión es en consecuencia: ns = 0.727 mol y la molalidad será: 0.727 mol de soluto m= = 0.727 molal 1 kg de agua El punto de ebullición de la solución es: De acuerdo a la relación (3.7) ∆Te = Kem ∆Te = 0.52 °C/m ∗ 0.727 m = 0.38 °C La temperatura de ebullición de la solución es: Te = (100 + 0.38) °C = 100.38 °C 3.10 OSMOSIS Y PRESIÓN OSMÓTICA L a osmosis es el proceso espontáneo por el cual las moléculas del disolvente atraviesan una membrana semipermeable de una solución de menor concentración de soluto hacia una solución con mayor concentración de mayor soluto COMPENDIOS DE QUÍMICA GENERAL PARA ESTUDIANTES DE CIENCIAS E INGENIERÍA
  • 22. 78 SOLUCIONES Y PROPIEDADES COLIGATIVAS Para definir la presión osmótica es conveniente considerar el sistema de la figura 2.12. El cual muestra un experimento en una cámara de presión osmótica. Rebalse de la solución Figura 3.14 Solución de azúcar Las moléculas de Agua Figura 3.13 azúcar no pueden atravesar la membrana H = π = ρgh Membrana Las moléculas de agua Figura 3.12 atraviesan la membrana Disolvente puro (Agua) Figura 3.12 - 3.14 Experimento de ósmosis. El agua pasa a través de la membrana a la solución de azúcar en el compartimiento del sistema. El flujo de agua cesa cuando el líquido en el embudo ejerce hacia abajo una presión suficiente (la presión osmótica). a) La figura 3.12 muestra el inicio de la determinación de la presión osmótica, los niveles de solución del lado izquierdo y del disolvente del lado derecho son iguales. b) Después del inicio del experimento, las moléculas del disolvente tienden a fluir hacia la solución, entonces observamos rebalse de la solución como era de esperarse, como muestra la figura 3.13. c) Para evitar el rebalse instalemos un tubo en la cámara de la disolución; durante cierto tiempo de iniciado el experimento, el flujo de la moléculas hacia la solución cesa y el sistema alcanza el equilibrio cuando el menisco se ha elevado hasta una determinada altura, como se Fig. 2.13 puede observar en la figura 3.14. En estas condiciones de equilibrio, la solución se encuentra bajo una presión hidrostática mayor que el disolvente puro. La altura del menisco multiplicada por la densidad de la solución y la aceleración de la gravedad, dá la presión adicional sobre la solución y ésta es la presión osmótica π. Por la medición experimental realizada en soluciones diluidas de concentración conocida, se sabe que la relación entre la presión osmótica y la concentración está dada simplemente por: π = cRT (3.15) Donde c es la concentración de la disolución en moles/litro, R es la constante universal de los gases y T es la temperatura absoluta. La presión osmótica es proporcional a la temperatura absoluta porque afecta el número de colisiones del disolvente con la membrana por unidad de tiempo, también es proporcional a la concentración molar, ya que ésta afecta a la diferencia en el número de moléculas del disolvente que chocan contra la membrana de cada lado. Ejemplo 3.18.- Estime el peso molecular de una macromolécula biológica si se disuelve una muestra de 0.194 gramos en 82.4 ml de benceno y se observa una presión osmótica de 14.20 torr a 25 °C. Solución: De la ecuación (3.15) π = cRT: π 14.20 mmHg −4 mol c= = = 7.64 ∗ 10 COMPENDIOS DE QUÍMICA GENERAL PARA ESTUDIANTES DE CIENCIAS E INGENIERÍA mmHg −  RT  62.4 ∗ 298K K − mol
  • 23. SOLUCIONES Y PROPIEDADES COLIGATIVAS 79 Considerando que el volumen de la solución es 82.4 ml, se puede determinar el número de moles de la macromolécula biológica: mol 7 ∗ 10 − 4 ∗ 0.0824 = 5.768 ∗ 10 − 5 mol  El peso molecular de dicha sustancia biológica es: m 0.194g g M= = −5 = 3363.38 n 5.768x10 mol 3.10.1 Ósmosis Inversa E l6 proceso de ósmosis inversa se ha aplicado al problema de la purificación del agua, en particular el método se ha utilizado para desalar el agua del océano (esto es para eliminar lar sales del agua de mar y obtener agua que se pueda beber o que sea industrialmente utilizable). En la ósmosis normal, el disolvente fluye a través de una membrana de una solución diluida a una solución mas concentrada, el proceso de ósmosis se puede detener. Figura 3.15 Sistema de desalación que utiliza la ósmosis inversa. Cuando el agua del océano se somete a una presión mayor que su presión osmótica, el agua pura pasa a través de una serie de membranas y dejan detrás una solución de sal más concentrada. Si se aplica una presión aún mayor, el proceso osmótico puede ser invertido, entonces el disolvente fluye de la solución concentrada (que pueda ser agua del océano), a través de una membrana, a una solución más diluida (que pueda ser agua mas o menos pura). En la figura 3.15 se presenta un sistema que utiliza ósmosis inversa para desalar el agua del océano. 3.11 Propiedades Coligativas de las Soluciones Ionicas P ara explicar las propiedades coligativas de las soluciones iónicas, debe aceptarse que la concentración total de iones, es mas importante que la concentración de una sustancia iónica. Por ejemplo, la depresión de congelación de una solución 0.100 m de cloruro de sodio es casi el doble del de una solución 0.100 m de glucosa. Puede explicarse esto diciendo que el cloruro de sodio se disuelve en agua para formar los iones Na+ y Cl-. Cada unidad fórmula de NaCl da dos partículas. Para cada propiedad coligativa de las soluciones iónicas se pueden considerar tres valores: el valor calculado ∆Tm, suponiendo el compuesto formado por moléculas; el valor real, ∆T, bastante mayor, encontrado experimentalmente; y el valor ideal ∆Ti, mayor todavía, que puede también calcularse al suponer el compuesto formado por iones que se comportasen en la disolución como si fueran partículas neutras.. 3.11.1 Factor de van’t Hoff La relación: ΔT i= (3.16) ΔTm 6 COMPENDIOS DE QUÍMICA GENERAL PARA ESTUDIANTES DE CIENCIAS E INGENIERÍA
  • 24. 80 SOLUCIONES Y PROPIEDADES COLIGATIVAS donde i es mayor a la unidad, se conoce como factor de van’Hoff. Cuando se suponía que los electrolitos estaban constituidos por moléculas se aceptó que una fracción de las mismas se disociaba en iones y que se establecía un equilibrio entre las moléculas sin disociar y los iones formados. La fracción de las moléculas ionizadas o disociadas se denomina grado de disociación. Para los electrolitos fuertes se establece actualmente la relación: ΔT =g (3.17) ΔTi donde g es menor que la unidad y que se conoce como coeficiente osmótico. Cuanto mas se acerca a la unidad el valor de g mayor es el comportamiento ideal de los iones en la disolución iónica. Si una supuesta molécula del electrolito se disocia en ν iones, es evidente que: ∆Ti = ν∆Tm , y por tanto; i g= (3.18) υ En el caso de los electrolitos débiles, si α es el grado de ionización, y una molécula forma realmente ν iones, 1 mol del electrolito dará lugar a να moles y quedaran sin ionizar ( 1 - α) moles, por lo cual, en vez de un mol de compuesto tendremos: [1+(ν - 1)α] moles de partículas, y puesto que cualquier propiedad coligativa es i veces mayor que el valor teórico correspondiente al numero de moléculas o moles disueltos, tendremos i = 1 + (ν - 1)α (3.19) y i −1 α= (3.20) υ −1 Esta expresión se aplica también corrientemente a los electrolitos fuertes, aunque ya se ha indicado que es incorrecto hablar en ellos de grado de disociación. Para no romper con esta costumbre nos referiremos nosotros a un grado de disociación aparente para explicar el comportamiento de los electrolitos fuertes. El factor de van’t Hoff i para soluciones iónicas en las expresiones de las propiedades coligativas se puede escribir: Descenso de la presión de vapor md P = P° Md md ms (3.21) +i Ms Ms Aumento del punto de ebullición ∆Te = iKem (3.22) Descenso del punto de fusión COMPENDIOS DE QUÍMICA GENERAL PARA ESTUDIANTES DE CIENCIAS E INGENIERÍA
  • 25. SOLUCIONES Y PROPIEDADES COLIGATIVAS 81 ∆Tc = iKcm (3.23) Presión osmótica π = icRT (3.24) Ejemplo 3.19.- A 100 °C la presión de vapor de una disolución de 10.0 g de nitrato cálcico en 150 g de agua es de 746.8 mmHg. Hallar el grado de disociación aparente del nitrato cálcico en esta disolución. Solución: Para hallar el grado de disociación aparente (véase ecuación 3.15) del nitrato cálcico (electrolito fuerte) debemos determinar el coeficiente de van’t Hoff a partir de la ecuación (3.16) y el número de iones ν Ca(NO3)2 = [ Ca++ + 2NO3-] ν=3 Así que, de la ecuación (3.16): md P = P° Md md ms +i Ms Ms reemplazando datos: 150 746.8 = 760 18 150 10 +i 18 164 efectuando operaciones con una máquina electrónica: 8.33 0.982 = 8.33 + 0.0608 i i = 2.42 Aplicando la expresión (3.15) i − 1 2.42 − 1 α= = = 0.71 υ −1 3−1 Ejemplo 3.20.- Una disolución 0.065 molal de cloruro cálcico empieza a congelar a – 0.322 °C, hallar: a) el grado de disociación del cloruro cálcico y b) la concentración de los iones cloruro y de los iones calcio en la disolución. Suponer la densidad de la disolución igual a la unidad. Kc(H2O = 1.86 °C/mol) Solución: a) Para hallar el grado de disociación haremos uso de la ecuación (2.15), sin embargo es preciso evaluar i y ν. El cloruro de calcio se disocia: CaCl2 = [ Ca++ + 2Cl- ] ν=3 De la ecuación (3.18) ∆Tc = iKcm: Despejamos i: ΔTc [ 0 − (−0.322) ] i= = = 2.66 Kc ∗ m 1.86 ∗ 0.065 COMPENDIOS DE QUÍMICA GENERAL PARA ESTUDIANTES DE CIENCIAS E INGENIERÍA
  • 26. 82 SOLUCIONES Y PROPIEDADES COLIGATIVAS Por consiguiente 2.66 − 1 α= ∗ 100 = 0.83 3 −1 b) Para hallar la concentración molar de los iones, determinaremos en primera instancia la concentración del cloruro de calcio: La masa total de la solución se puede calcular: 0.065 mol CaCl 2 111 g CaCl 2 7.215 g CaCl 2 ∗ = 1000 g H 2 O 1 mol CaCl 2 1000 g H 2 O Es decir la masa de la solución es: msolución = 1000 g puesto que la densidad es 1.0 g/ml El volumen de la solución es: V = 1000 ml = 1.000  La concentración molar de la solución iónica de CaCl2 es: 0.065 mol de CaCl 2 M= = 0.065 molar 1.000  y las concentraciones de los iones determinamos a partir de : 0.065 ∗ 0.83 = 0.0540 CaCl2 = Ca++ + 2Cl- 1 mol 1 mol 2 mol [ Ca++ ]= 0.0540 molar, la relación es 1:1 [ Cl- ] = 2 ( 0.0540) = 0.108 molar la relación es 1:2 PROBLEMAS RESUELTOS 3.1 Una solución de ácido sulfúrico tiene densidad de 1.84 g/ml y contiene 98% en masa de H2SO4. ¿Qué volumen ocuparán 200 g de H2SO4 puro? Solución: Se puede considerar los conceptos básicos de factores de conversión: 100 g de solución 1 cm 3 de solución 200 g H 2SO 4 ∗ ∗ = 110.91 cm 3 de solución 98 g H 2SO 4 1.84 g de solución 110.91cm 3 = VH 2 O + VH 2 SO 4 (1) El volumen de agua es: 100 g de solución 2 g de H 2 O 1 cm 3 H 2 O 200 g H 2SO 4 ∗ ∗ ∗ = 4.082 cm 3 H 2 O 98 g H 2SO 4 100 g de solución 1 g H 2 O En consecuencia, considerando la ecuación (1), el volumen de H2SO4 puro es: VH 2SO 4 = 110.91 − 4.082 = 106.82cm 3 H 2SO 4 COMPENDIOS DE QUÍMICA GENERAL PARA ESTUDIANTES DE CIENCIAS E INGENIERÍA
  • 27. SOLUCIONES Y PROPIEDADES COLIGATIVAS 83 3.2 Calcule la molalidad, fracciones molares de soluto y disolvente, y la molaridad de las siguientes soluciones: a) una solución acuosa de H2SO4 que es 50 % en masa y tiene una densidad de 1.4 g/ml, b) una solución acuosa de sacarosa, C12H22O11, que es 19 por ciento sacarosa en masa, y tiene una densidad de 1.08 g/ml, y c) una solución compuesta de 24.4 g de NaOH y 97.6 de H2O con un volumen de 100 ml. Solución: a) La molalidad de la solución de H2SO4 se puede determinar a partir de la composición centesimal, vale decir: 50 g H 2SO 4 1000 g H 2 O 1 mol H 2SO 4 m= ∗ ∗ = m = 10.2 molal 50 g H 2O 1 kg H 2 O 98 g H 2SO 4 La fracción molar se determina calculando previamente los moles del soluto y disolvente: 1 mol H 2SO 4 50 g H 2SO 4 ∗ = 0.510 mol H 2SO 4 98 g H 2SO 4 1 mol H 2 O 50 g H 2 O ∗ = 2.78 mol H 2O 18 g H 2 O La fracción molar será entonces : 0.510 X H 2 SO 4 = = 0.155 3.290 X H2O = 1 − 0.155 = 0.845 La molaridad se calcula considerando la densidad de la solución g Solución 50 g H 2SO 4 1 mol H 2SO 4 1000 mlSol. M = 1.4 ∗ ∗ ∗ = 7.14 molar ml Solución 100 g Solución 98 g H 2SO 4 1 Solución b) La molalidad de la solución de C12H22O11, se puede determinar a partir de la composición centesimal, vale decir: 19 g C12 H 22 O11 1000 g H 2O 1 mol C12 H 22 O11 m= ∗ ∗ = 0.6859 molal 81 g H 2 O 1 kg H 2 O 342 g C12 H 22 O11 La fracción molar se determina calculando previamente los moles del soluto y disolvente: 1 mol C12 H 22 O11 n C12 H 22 O11 = 19 g C12 H 22O11 ∗ = n C12 H 22 O11 = 0.0555 mol C12 H 22 O11 342 g C12 H 22 O11 1 mol H 2O 81 g H 2 O ∗ = 4.50 mol H 2 O 18 g H 2 O La fracción molar será entonces : 0.0555 X C12 H 22 O11 = = 0.012 4.5555 X H2O = 1 − 0.012 = 0.988 COMPENDIOS DE QUÍMICA GENERAL PARA ESTUDIANTES DE CIENCIAS E INGENIERÍA
  • 28. 84 SOLUCIONES Y PROPIEDADES COLIGATIVAS La molaridad se calcula considerando la densidad de la solución g Solución 19gC12 H 22 O11 1 mol C12 H 22 O11 M =1.08 ∗ ∗ ∗ ml Solución 100 g Solución 342 g C12 H 22 O11 1000 ml Solución = 0.6 molar 1 Solución 3.3 Una solución acuosa de cloruro de sodio que contiene 8.00 g de NaCl por 100 g de solución posee una densidad de 1.054 g/ml a la temperatura de 25ºC. Determine: a) la concentración molar y b) la concentración molal de la solución. Solución: Nº moles de soluto a) 8 g NaCl/100 g sol. Molaridad = 1 litro de solución ρ = 1.054 g/ml moles de soluto : m 8g n= = = 0.137 mol NaCl M 58.5 g/mol -Cálculo del volumen en litros de solución: 1 ml de solución V = 100 g ∗ = 94.88 ml 1.054 g V= 0.0949 litros Molaridad : 0.137 mol NaCl M= = 1.44 molar 0.0949  Nº de moles de soluto b) Molalidad = kg de solvente Cálculo de la masa en kilogramos de solvente Masa del solvente = masa (solución) - masa (soluto) = 100 g - 8 g = 92 g de solvente Kg de solvente = 92g = 0.092 Kg 0.137 mol de soluto Molalidad = = 1.49 molal 0.092 kg de solvente 3.4 Se prepara 1.5 litros de una solución de H2O y C2H5OH, cuya densidad relativa resulta 0.947 en una proporción volumétrica de 3:1 respectivamente. Determinar: a) La densidad del alcohol etílico. b) La fracción molar del alcohol etílico. c) La molaridad. d) La molalidad. Solución: Datos: Vsolución = 1.5  ρsolución = 0.947 kg/ %VH2O = 75% %VC2H5OH = 25% a) VH2O = 1.5  ∗ 0.75 = 1.125  COMPENDIOS DE QUÍMICA GENERAL PARA ESTUDIANTES DE CIENCIAS E INGENIERÍA
  • 29. SOLUCIONES Y PROPIEDADES COLIGATIVAS 85 VC2H5OH = 1.5  ∗ 0.25 = 0.375  kg m sol m sol = ρ sol ∗ Vsol = 0.947 ∗ 1.5 ρ sol =  Vsol m sol = 1.4205 kg En consecuencia la masa del alcohol etílico es: mC2H5OH = msol – mH2O mC2H5OH =1.4305 – 1.125 = 0.2955 kg mC2H5OH = 295.5 g La densidad del alcohol etílico es: m 295.5g ρ= = = 0.788g/ml V 375ml n C H OH b) χ C 2 H 5 OH = 2 5 nT 295.5 g n C 2 H 5 OH = = 6.424 mol g 46 mol 1125 g n H 2O = = 62.5 mol g 18 mol nT = 6.424+62.5= 68.924 mol 6.424mol χ C2 H 5OH = = 0.093 68.924mol c) n soluto 6.424 C 2 H 5 OHmol M= = = 4.28 molar  solución 1.5 solución d) n soluto 6.424 molC 2 H 5 OH m= = = 5.71 molal kg dilsolvente 1.125 kg H 2 O 3.5 ( 20 puntos, 20 min) La densidad relativa de una disolución acuosa de cloruro de potasio que contiene 24.6 g KCl es de 1.131 a 21 ºC, mientras que la densidad relativa del cloruro de potasio sólido, a la misma temperatura es 1.984. Calcular: a) la molaridad, b) la normalidad, c) la molalidad, d) la fracción molar, y e) el tanto por ciento en volumen. (K = 39; Cl = 35.5) Solución: Datos: ρSolución= 1.131 g/ml; ρKCl= 1.984 g/ml; ρH2O= 1.0 g/ml; mKCl = 24.6 g mH2O = x m m KCl + m H 2O ρ solución = total = (1) Vtotal V KCl + V H 2O mH2O = VH2O (2) m 24.6 g V KCl = KCl = = 12.40ml (3) ρ KCl 1.984 Reemplazando en (1) COMPENDIOS DE QUÍMICA GENERAL PARA ESTUDIANTES DE CIENCIAS E INGENIERÍA
  • 30. 86 SOLUCIONES Y PROPIEDADES COLIGATIVAS 24.6 g + m H 2O 1.131 = 12.4 + m H 2O mH2O = 80.76 g VH2O = 80.76 ml Volumen total: Vsolución = 12.4 ml + 80.76 ml = 93.16 ml a) la molaridad es: 24.6 gKCl 1000ml 1molKCl ∗ ∗ = 3.54molar 93.16ml 1 74.5 gKCl b) La normalidad es: 24.6 gKCl 1000ml 1molKCl 1eq − gKCl ∗ ∗ ∗ = 3.54normal 93.16ml 1 74.5 gKCl 1molKCl c) La molalidad: 24.6 gKCl 1000 gH 2O 1molKCl ∗ ∗ = 4.09molal 80.76 gH 2O 1KgH 2O 74.5 gKCl d) La fracción molar: 1molKCl 1molH 2 O 24.6 gKCl ∗ = 0.33molKCl 80.76 gH 2 O ∗ = 4.49molH 2 O 74.5 gKCl 18 gH 2 O 0.33 4.49 X KCl = = 0.068 X H 2O = = 0.932 4.82 4.82 12.4ml e) El % en volumen: %V KCl = ∗ 100% = 13.31% 93.16ml 3.6.- Determinar: a) la concentración molal, b) la fracción molar y la molaridad de una solución de ácido sulfúrico del 50% en masa y cuya densidad es 1.4 g/ml. Solución: a) La molalidad de la solución de H2SO4 se puede determinar a partir de la composición centesimal, vale decir: 50 g H 2SO 4 1000 g H 2O 1 mol H 2SO 4 m= ∗ ∗ = 10.2 molal 50 g H 2 O 1 kg H 2 O 98 g H 2SO 4 La fracción molar se determina calculando previamente los moles del soluto y disolvente: 1 mol H 2SO 4 50 g H 2SO 4 ∗ = 0.510 mol H 2SO 4 98 g H 2SO 4 1 mol H 2 O 50 g H 2 O ∗ = 2.78 mol H 2 O 18 g H 2 O La fracción molar será entonces : 0.510 X H 2SO 4 = = 0.155 3.290 X H2O = 1 − 0.155 = 0.845 La molaridad se calcula considerando la densidad de la solución g Solución 50 g H 2SO 4 1 mol H 2SO 4 1000 ml Solución M = 1.4 ∗ ∗ ∗ = 7.14 molar COMPENDIOSml Solución 100 g Solución 98 g H 2SO 4 DE QUÍMICA GENERAL PARA ESTUDIANTES Solución1  DE CIENCIAS E INGENIERÍA
  • 31. SOLUCIONES Y PROPIEDADES COLIGATIVAS 87 3.7 A 100 ml de una disolución de ácido sulfúrico del 96% en masa y de densidad relativa 1.84, se añadieron 400 ml de agua, obteniéndose una solución de peso específico relativo igual a 1.22. En la solución resultante determinar: a) la concentración en tanto por ciento en masa. b) La concentración molar c) La concentración molal d) La normalidad e) La fracción molar Solución: El lector debe hacer énfasis en esquemas de manera que le permitan ver objetivamente el problema: H2O V = 100 ml V = ¿? 96%H2SO4 %H2SO4 =¿? ρ = 1.84 ρ = 1.22 A 100 ml de una solución de ácido sulfúrico se añade 400 ml de agua, a esta operación se denomina dilución, que obviamente modificará la concentración y la densidad de la solución. Para resolver el problema calcularemos el volumen de agua de la solución original para luego sumar a los 400 ml de agua que se añadieron, a partir de este dato la masa de agua de modo que sumados a la masa de ácido sulfúrico puro y considerando la densidad de la solución resultante se podrá determinar el volumen de la solución resultante. Determinación de la masa de H2O y H2SO4 de la solución original 1.84 g de sol. 100 ml de sol. ∗ = 184 g de solución 1 ml de sol. 96 g H 2SO 4 184 g de solución ∗ = 176.64 g H 2SO 4 la masa de agua en la 100g de solución solución resultante m agua = 184 g − 176.64 g = 7.36 g H 2 O será: mH2O = (7.36 + 400) g = 407.36 g H2O De acuerdo a algunas estrategias recomendadas en el presente texto es fundamental conocer la concentración de la solución resultante en tanto por ciento: a) El tanto por ciento del ácido sulfúrico se determina a partir de: m H2SO4 = 176.64 g m H2O = 407.36 g La masa total es: (176.64 + 407.36) g = 584 g 176.64 %H 2SO 4 = ∗ 100% = 43.36% 407.36 COMPENDIOS DE QUÍMICA GENERAL PARA ESTUDIANTES DE CIENCIAS E INGENIERÍA